2010-2019年考研英语二历年真题及答案解析


2023年12月23日发(作者:盏的组词)

2010——2019年考研英语二真题及答案

2010考研英语二真题及答案

Section I Use of English

Directions:

Read the following passage. For each numbered blank there are four choices marked A, B, C and D.

Choose the best one and mark your answers on ANSWER SHEET l. (10 points)

The outbreak of swine flu that was first detected in Mexico was declared a global epidemic on June 11,

2009. It is the first worldwide epidemic_____1_____ by the World Health Organization in 41 years.

The heightened alert _____2_____an emergency meeting with flu experts in Geneva that convened

after a sharp rise in cases in Australia, and rising_____3_____in Britain, Japan, Chile and elsewhere.

But the epidemic is "_____4_____" in severity, according to Margaret Chan, the organization's

director general, _____5_____ the overwhelming majority of patients experiencing only mild symptoms

and a full recovery, often in the _____6_____ of any medical treatment.

The outbreak came to global_____7_____in late April 2009, when Mexican authorities noticed an

unusually large number of hospitalizations and deaths_____8_____healthy adults. As much of Mexico City

shut down at the height of a panic, cases began to _____9_____in New York City, the southwestern United

States and around the world.

In the United States, new cases seemed to fade_____10_____warmer weather arrived. But in late

September 2009, officials reported there was _____11_____flu activity in almost every state and that

virtually all the_____12_____tested are the new swine flu, also known as (A) H1N1, not seasonal flu. In

the U.S., it has_____13_____more than one million people, and caused more than 600 deaths and more

than 6,000 hospitalizations.

Federal health officials_____14_____Tamiflu for children from the national stockpile and

began_____15_____orders from the states for the new swine flu vaccine. The new vaccine, which is

different from the annual flu vaccine, is ____16_____ ahead of expectations. More than three million doses

were to be made available in early October 2009, though most of those _____17_____doses were of the

FluMist nasal spray type, which is not_____18_____for pregnant women, people over 50 or those with

breathing difficulties, heart disease or several other _____19_____. But it was still possible to vaccinate

people in other high-risk group: health care workers, people _____20_____infants and healthy young

people.

1 [A] criticized

2 [A] proceeded

3 [A] digits

4 [A] moderate

[B] appointed

[B] activated

[B] numbers

[B] normal

[C]commented

[C] followed

[C] amounts

[C] unusual

1

[D] designated

[D] prompted

[D] sums

[D] extreme

[D] by 5 [A] with [B] in [C] from

2010——2019年考研英语二真题及答案

6 [A] progress

7 [A] reality

8. [A]over

10 [A] as

9 [A] stay up

[B] absence

[B] for

[B] if

[C] presence

[C] concept

[C] among

[C] fill up

[C] unless

[C] significant

[C] patterns

[C] injected

[C] taking

[C] reliable

[C] relieved

[D] favor

[D] notice

[D] to

[D] cover up

[D] until

[D]magnificent

[D] samples

[D] infected

[D] remained

[D] giving

[D] applicable

[D] initial

[D] introduced

[D] sufferings

[D] warding off

[B] phenomenon

[B] crop up

[B] enormous

[B] examples

[B] immerse

[B] relayed

[B] available

[B] principal

[B] restricted

[B] issues

[B] caring for

11 [A] excessive

12 [A]categories

13 [A] imparted

14 [A] released

16 [A] feasible

17 [A] prevalent

15 [A] placing [B] delivering

[C] innovative

[C] agonies

18 [A] presented

19 [A] problems

20 [A] involved in

[C] recommended

[C] concerned with

Section Ⅱ Reading comprehension

Part A

Text1

The longest bull run in a century of art-market history ended on a dramatic note with a sale of 56

works by Damien Hirst, “Beautiful Inside My Head Forever”,at Sotheby’s in London on September 15th

2008. All but two pieces sold, fetching more than £70m, a record for a sale by a single artist. It was a last

victory. As the auctioneer called out bids, in New York one of the oldest banks on Wall Street, Lehman

Brothers, filed for bankruptcy.

The world art market had already been losing momentum for a while after rising bewilderingly since

2003. At its peak in 2007 it was worth some $65 billion, reckons Clare McAndrew, founder of Arts

Economics, a research firm—double the figure five years earlier. Since then it may have come down to $50

billion. But the market generates interest far beyond its size because it brings together great wealth,

enormous egos, greed, passion and controversy in a way matched by few other industries.

In the weeks and months that followed Mr Hirst’s sale, spending of any sort became deeply

unfashionable, especially in New York, where the bail-out of the banks coincided with the loss of thousands

of jobs and the financial demise of many art-buying investors. In the art world that meant collectors stayed

away from galleries and salerooms. Sales of contemporary art fell by two-thirds, and in the most

overheated sector—for Chinese contemporary art—they were down by nearly 90% in the year to

November 2008. Within weeks the world’s two biggest auction houses, Sotheby’s and Christie’s, had to pay

out nearly $200m in guarantees to clients who had placed works for sale with them.

The current downturn in the art market is the worst since the Japanese stopped buying Impressionists

2

2010——2019年考研英语二真题及答案

at the end of 1989, a move that started the most serious contraction in the market since the Second World

War. This time experts reckon that prices are about 40% down on their peak on average, though some have

been far more fluctuant. But Edward Dolman, Christie’s chief executive, says: “I’m pretty confident we’re

at the bottom.”

What makes this slump different from the last, he says, is that there are still buyers in the market,

whereas in the early 1990s, when interest rates were high, there was no demand even though many

collectors wanted to sell. Christie’s revenues in the first half of 2009 were still higher than in the first half

of 2006. Almost everyone who was interviewed for this special report said that the biggest problem at the

moment is not a lack of demand but a lack of good work to sell. The three Ds—death, debt and

divorce—still deliver works of art to the market. But anyone who does not have to sell is keeping away,

waiting for confidence to return.

the first paragraph, Damien Hirst's sale was referred to as “a last victory” because ____.

A. the art market had witnessed a succession of victories

B. the auctioneer finally got the two pieces at the highest bids

C. Beautiful Inside My Head Forever won over all masterpieces

D. it was successfully made just before the world financial crisis

saying “spending of any sort became deeply unfashionable”(Line 1-2,Para.3),the author

suggests that_____.

A. collectors were no longer actively involved in art-market auctions

B .people stopped every kind of spending and stayed away from galleries

C. art collection as a fashion had lost its appeal to a great extent

D .works of art in general had gone out of fashion so they were not worth buying

23. Which of the following statements is NOT true?

A .Sales of contemporary art fell dramatically from 2007 to 2008.

B. The art market surpassed many other industries in momentum.

C. The market generally went downward in various ways.

D. Some art dealers were awaiting better chances to come.

24. The three Ds mentioned in the last paragraph are ____

A. auction houses ' favorites

B. contemporary trends

C. factors promoting artwork circulation

D. styles representing impressionists

25. The most appropriate title for this text could be ___

A. Fluctuation of Art Prices

B. Up-to-date Art Auctions

C. Art Market in Decline

D. Shifted Interest in Arts

3

2010——2019年考研英语二真题及答案

Text2

I was addressing a small gathering in a suburban Virginia living room—a women's group that had

invited men to join them. Throughout the evening one man had been particularly talkative, frequently

offering ideas and anecdotes, while his wife sat silently beside him on the couch. Toward the end of the

evening I commented that women frequently complain that their husbands don't talk to them. This man

quickly nodded in agreement. He gestured toward his wife and said, "She's the talker in our family." The

room burst into laughter; the man looked puzzled and hurt. "It's true," he explained. "When I come home

from work, I have nothing to say. If she didn't keep the conversation going, we'd spend the whole evening

in silence."

This episode crystallizes the irony that although American men tend to talk more than women in

public situations, they often talk less at home. And this pattern is wreaking havoc with marriage.

The pattern was observed by political scientist Andrew Hacker in the late 1970s. Sociologist Catherine

Kohler Riessman reports in her new book "Divorce Talk" that most of the women she interviewed—but

only a few of the men—gave lack of communication as the reason for their divorces. Given the current

divorce rate of nearly 50 percent,that amounts to millions of cases in the United States every year —a

virtual epidemic of failed conversation.

In my own research complaints from women about their husbands most often focused not on tangible

inequities such as having given up the chance for a career to accompany a husband to his or doing far more

than their share of daily life-support work like cleaning, cooking, social arrangements and errands. Instead

they focused on communication: "He doesn't listen to me." "He doesn't talk to me." I found as Hacker

observed years before that most wives want their husbands to be first and foremost conversational partners

but few husbands share this expectation of their wives.

In short the image that best represents the current crisis is the stereotypical cartoon scene of a man

sitting at the breakfast table with a newspaper held up in front of his face, while a woman glares at the back

of it, wanting to talk.

26. What is most wives' main expectation of their husbands?

A. Talking to them.

B. Trusting them.

C. Supporting their careers.

D. Sharing housework.

27. Judging from the context, the phrase “wreaking havoc”(Line 3,Para.2)most probably means ___ .

A. generating motivation.

B. exerting influence

C. causing damage

D. creating pressure

28. All of the following are true EXCEPT_______

A. men tend to talk more in public than women

B. nearly 50 percent of recent divorces are caused by failed conversation

4

2010——2019年考研英语二真题及答案

C. women attach much importance to communication between couples

D. a female tends to be more talkative at home than her spouse

29. Which of the following can best summarize the main idea of this text?

A. The moral decaying deserves more research by sociologists.

B. Marriage break-up stems from sex inequalities.

C. Husband and wife have different expectations from their marriage.

D. Conversational patterns between man and wife are different.

30. In the following part immediately after this text, the author will most probably focus on ______

A. a vivid account of the new book Divorce Talk

B. a detailed description of the stereotypical cartoon

C. other possible reasons for a high divorce rate in the U.S.

D. a brief introduction to the political scientist Andrew Hacker

Text 3

Over the past decade, many companies had perfected the art of creating automatic behaviors — habits

— among consumers. These habits have helped companies earn billions of dollars when customers eat

snacks, apply lotions and wipe counters almost without thinking, often in response to a carefully designed

set of daily cues.

“There are fundamental public health problems, like dirty hands instead of a soap habit, that remain

killers only because we can’t figure out how to change people’s habits,” Dr. Curtis said. “We wanted to

learn from private industry how to create new behaviors that happen automatically.”

The companies that Dr. Curtis turned to — Procter & Gamble, Colgate-Palmolive and Unilever — had

invested hundreds of millions of dollars finding the subtle cues in consumers’ lives that corporations could

use to introduce new routines.

If you look hard enough, you’ll find that many of the products we use every day — chewing gums, skin

moisturizers, disinfecting wipes, air fresheners, water purifiers, health snacks, antiperspirants, colognes,

teeth whiteners, fabric softeners, vitamins— are results of manufactured habits. A century ago, few people

regularly brushed their teeth multiple times a day. Today, because of canny advertising and public health

campaigns, many Americans habitually give their pearly whites a cavity-preventing scrub twice a day, often

with Colgate, Crest or one of the other brands.

A few decades ago, many people didn’t drink water outside of a meal. Then beverage companies

started bottling the production of far-off springs, and now office workers unthinkingly sip bottled water all

day long. Chewing gum, once bought primarily by adolescent boys, is now featured in commercials as a

breath freshener and teeth cleanser for use after a meal. Skin moisturizers are advertised as part of morning

beauty rituals, slipped in between hair brushing and putting on makeup.

“Our products succeed when they become part of daily or weekly patterns,” said Carol Berning, a

consumer psychologist who recently retired from Procter & Gamble, the company that sold $76 billion of

Tide, Crest and other products last year. “Creating positive habits is a huge part of improving our

consumers’ lives, and it’s essential to making new products commercially viable.”

5

2010——2019年考研英语二真题及答案

Through experiments and observation, social scientists like Dr. Berning have learned that there is

power in tying certain behaviors to habitual cues through relentless advertising. As this new science of

habit has emerged, controversies have erupted when the tactics have been used to sell questionable beauty

creams or unhealthy foods.

31. According to Dr. Curtis, habits like hand washing with soap________.

[A] should be further cultivated

[B] should be changed gradually

[C] are deeply rooted in history

[D] are basically private concerns

32. Bottled water, chewing gun and skin moisturizers are mentioned in Paragraph 5 so as to____

[A] reveal their impact on people’s habits

[B] show the urgent need of daily necessities

[C] indicate their effect on people’s buying power

[D] manifest the significant role of good habits

33. Which of the following does NOT belong to products that help create people’s habits?

[A]Tide [B] Crest

[C] Colgate [D] Unilever

34. From the text we know that some of consumer’s habits are developed due to _____

[A]perfected art of products [B]automatic behavior creation

[C]commercial promotions [D]scientific experiments

35. The author’s attitude toward the influence of advertisement on people’s habits is____

[A] indifferent [B] negative

[C] positive [D] biased

Text4

Many Americans regard the jury system as a concrete expression of crucial democratic values,

including the principles that all citizens who meet minimal qualifications of age and literacy are equally

competent to serve on juries; that jurors should be selected randomly from a representative cross section of

the community; that no citizen should be denied the right to serve on a jury on account of race, religion, sex,

or national origin; that defendants are entitled to trial by their peers; and that verdicts should represent the

conscience of the community and not just the letter of the law. The jury is also said to be the best surviving

example of direct rather than representative democracy. In a direct democracy, citizens take turns governing

themselves, rather than electing representatives to govern for them.

But as recently as in 1986, jury selection procedures conflicted with these democratic ideals. In some

states, for example, jury duty was limited to persons of supposedly superior intelligence, education, and

moral character. Although the Supreme Court of the United States had prohibited intentional racial

discrimination in jury selection as early as the 1880 case of Strauder v. West Virginia, the practice of

selecting so-called elite or blue-ribbon juries provided a convenient way around this and other

antidiscrimination laws.

6

2010——2019年考研英语二真题及答案

The system also failed to regularly include women on juries until the mid-20th century. Although

women first served on state juries in Utah in 1898, it was not until the 1940s that a majority of states made

women eligible for jury duty. Even then several states automatically exempted women from jury duty

unless they personally asked to have their names included on the jury list. This practice was justified by the

claim that women were needed at home, and it kept juries unrepresentative of women through the 1960s.

In 1968, the Congress of the United States passed the Jury Selection and Service Act, ushering in a

new era of democratic reforms for the jury. This law abolished special educational requirements for federal

jurors and required them to be selected at random from a cross section of the entire community. In the

landmark 1975 decision Taylor vs. Louisiana, the Supreme Court extended the requirement that juries be

representative of all parts of the community to the state level. The Taylor decision also declared sex

discrimination in jury selection to be unconstitutional and ordered states to use the same procedures for

selecting male and female jurors.

36. From the principles of the US jury system, we learn that ______

[A]both liberate and illiterate people can serve on juries

[B]defendants are immune from trial by their peers

[C]no age limit should be imposed for jury service

[D]judgment should consider the opinion of the public

37. The practice of selecting so-called elite jurors prior to 1968 showed_____

[A]the inadequacy of antidiscrimination laws

[B]the prevalent discrimination against certain races

[C]the conflicting ideals in jury selection procedures

[D]the arrogance common among the Supreme Court justices

38. Even in the 1960s, women were seldom on the jury list in some states because_____

[A]they were automatically banned by state laws

[B]they fell far short of the required qualifications

[C]they were supposed to perform domestic duties

[D]they tended to evade public engagement

39. After the Jury Selection and Service Act was passed.___

[A] sex discrimination in jury selection was unconstitutional and had to be abolished

[B] educational requirements became less rigid in the selection of federal jurors

[C] jurors at the state level ought to be representative of the entire community

[D] states ought to conform to the federal court in reforming the jury system

40. In discussing the US jury system, the text centers on_______

[A]its nature and problems

[B]its characteristics and tradition

[C]its problems and their solutions

[D]its tradition and development

7

2010——2019年考研英语二真题及答案

Section Ⅲ Translation

ions:

In this section there is a text in English .Translate it into Chinese. Write your translation on ANSWER

SHEET2.(15points)

“Suatainability” has become apopular word these days, but to Ted Ning, the concept will always have

personal meaning. Having endured apainful period of unsustainability in his own life made itclear to him

that sustainability-oriented values must be expressed though everyday action and choice。Ning recalls

spending aconfusing year in the late 1990s selling insurance. He’d been though the dot-com boom and

burst and,desperate for ajob,signed on with a Boulder agency。It didin’t go well. “It was a really had move

because that’s not my passion,” says Ning, whose dilemma about the job translated, predictably, into a lack

of sales. “I was miserable, I had so much anxiety that I would wake up in the middle of the night and stare

at the ceiling. I had no money and needed the job. Everyone said, ‘Just wait, you’ll trun the corner, give it

some time.’”

47. Directions:

You have just come back from the U.S. as a member of a Sino-American cultural exchange program. Write

a letter to your American colleague to

1) Express your thanks for his/her warm reception;

2) Welcome him/her to visit China in due course。

You should write about 100 words on ANSWER SHEET 2.

Do not sign your own name at the end of the letter. Use “Zhang Wei” instead。

Do not write your address. (10 points)

48. Directions:

In this section, you are asked to write an essay based on the following chart. In your writing, you should

1) Interpret the chart and

2) Give your comments。

You should write at least 150 words。

Write your essay on on ANSWER SHEET 2. (15 points)

2010年考研英语二真题答案:

1 [D] designated 2 [C] followed 3 [B] numbers 4 [A] moderate5 [A] with 6 [B] absence

7 [D] notice8. [C] among9 [B] crop up 10 [A] as 11 [C] significant 12 [D] samples

13 [D] infected14 [A] released 15 [C] taking16 [B] available

18 [C] recommended19 [A] problems20 [B] caring for

21 D 22 A 23 B 24 C 25 C 26 A 27 C 28 B 29 C 30 C

31 A 32 A 33 D 34 C 35 B 36 D 37 C 38 C 39 B 40 A

8

17 [D] initial

2010——2019年考研英语二真题及答案

翻译参考

“坚持不懈”如今已成一个流行词汇,但对TedNing而言,这个概念一直有个人含义,经历了一段痛苦松懈的个人生活,使他清楚面向以坚持不懈为导向的价值观,必须贯彻到每天的行动和选择中。

Ning回忆起20世纪90年代末期卖保险的那段迷茫时光,他通过蓬勃兴起的网络疯狂地工作,并且与Boulder代理机构签了约。

事情进展并不顺利,TedNing说到:“那真是个糟糕的选择,因为我对此没有激情,”可以预料,他把工作中的矛盾能解释为没有业务。Ning说:“我很痛苦渴望午夜起来盯着天花板,我没钱,我需要工作,每个人都说‘等吧,只要有耐心会好转的。’”

小作文

Dear xxx,

I would like to convey my heartfelt thanks to you for your kindness to receive me when I participated

in an exchange program in USA.

Your generous help made it possible that I had a very pleasant stay and a chance to know American cultures

better. Besides, I think it is an honor for me to make friends with you and I w ill cherish the goodwill you

showed to me wherever I go. I do hope that you will visit China one day, so that I could have the

opportunity to repay your kindness and refresh our friendship。

I feel obliged to thank you again。

Sincerely yours,

Zhang Wei

大作文范文:

In this chart, we can see the mobile phone subscriptions in developed countries have a steady and slight

increase from 1990 to 2007 and then remain constant in 2008. Meanwhile the mobile phone subscriptions

in developing countries have witnessed a slow increase from 1990 to 2004 and then a great surge from

2004 to 20007: the biggest surge happens from 2005 to 2006.

This chart reflects different developing modes of mobile phone industry in developed and developing

countries. The developed countries have a limited number of populations, most of whom are well-educated.

Therefore, the spreading of the mobile phone service is efficient and soon the market is saturated. Also at

the beginning the developed countries have more people who can afford this service. The developing

countries have a large population who keeps a large demand for mobile service. As the mobile phone

service becomes cheaper and cheaper, the increasing customers subscribe to benefit from this service。

As discussed above, it is not surprising to see this change. In my opinion, this trend that the number of

mobile-phone subscriptions is increasingly increasing will continue for a while in the future

新题型译文:

波音和空客都在吹嘘自己最新飞机787和A350的性能,巧妙的设计与轻质的材料当然使它们与众不同。但在斯坦福大学,由伊兰▪克鲁领导的一个研究小组提出,飞机的飞行路线可以改用更具仿生效应的路径来节省燃油,而且并不需要购买新的飞机。

克鲁教授称这个设想是受到鸟类的启发。1914年,一位名叫卡尔▪维塞尔斯伯格的德国研究员发表了一篇9

2010——2019年考研英语二真题及答案

具有重大意义的论文,科学家们从中得知,鸟类在飞行的时候排成V字、梯形或倒过来时,会节省体力。气流在划过鸟类双翼时会在其后面弯曲向上,这是一种叫上升流的现象。在上升流中飞行的鸟阻力减小,可以用更小的力气飞。彼得▪利萨曼以前是加州理工学院和南加州大学的航空专家,他认为由25只鸟组成的队伍会将飞行距离提升71%。

当把鸟类的特性应用到飞机上的时候,原理也相差无几。克鲁教授和他的团队模拟了三架分别从洛杉矶、旧金山和拉斯维加斯起飞的客机在犹他州集合,采用反V字形状并偶尔换一下位置以便所有飞机轮流利用最有利位置,最后飞到伦敦。他们发现飞机节省多达15%的燃油,相应的二氧化碳排放也少了很多,途中排放的氮氧化物减少约四分之一。

当然,一定要有完善的操作指南。要考虑安全问题,至少在感觉上来说是安全的。乘客们在换位时会不会不舒服呢?克鲁教授指出飞机间会相隔几海里,不会像红箭表演队(英国皇家空军特技表演队)那样间距小到令人窒息。乘客往窗外看也许都看不到另外的飞机。有关飞机间距是否符合空中管制条例暂且不谈,但国际民航组织的一个工作组已经考虑在新的操作方针中列入编队飞行的可能性。

现在仍需考虑天气情况对气流的影响,这会关系到编队飞行的效率。在有大量乱流的地区,飞机尾部的气流会消失更快,上升流效应就会消失。克鲁教授称,在这方面他们团队将要进行更多的研究。每架客机的起飞时间与目的地都不相同,要协调它们编队飞行很难。相反,货机和日常空军飞行更容易协调时间。

恰逢此时,美军已经着手这项研究。今年早些时候,美国国防部高级研究计划局宣布出资让波音公司研究编队飞行的计划,但项目到现在还没启动。有报道称二战时就有一些军机在燃油不足的情况下编队飞行,但利萨曼教授称那些都是杜撰的,他说:“我父亲是皇家空军飞行员,我的堂兄是一架在柏林坠毁的兰喀斯特式飞机的机长”,所以他应该知道。

2011年研究生入学考试英语二真题

Section I Use of English

Directions:Read the following text. Choose the best word(s) for each numbered black and mark A, B, C or

D on ANSWER SHEET 1. (10 points)

"The Internet affords anonymity to its users — a boon to privacy and freedom of speech. But that very

anonymity is also behind the explosion of cybercrime that has 1 across the Web.

Can privacy be preserved 2 bringing a semblance of safety and security to a world that seems

increasingly 3 ?

Last month, Howard Schmidt, the nation’s cyberczar, offered the Obama government a 4 to make

the Web a safer place — a “voluntary identify” system that would be the high-tech 5 of a physical key,

fingerprint and a photo ID card, all rolled 6 one. The system might use a smart identity card, or a

digital credential 7 to a specific computer, and would authenticate users at a range of online services.

The idea is to 8 a federation of private online identify systems. Users could 9 which system

to join, and only registered users whose identities have been authenticated could navigate those systems.

The approach contrasts with one that would require an Internet driver’s license 10 by the government.

Google and Microsoft are among companies that already have sign-on” systems that make it possible

for users to 11 just once but use many different services.

10

2010——2019年考研英语二真题及答案

12 , the approach would create a “walled garden” in safe “neighborhoods” and bright “streetlights” to

establish a sense of 13 community.

Mr. Schmidt described it as a “voluntary ecosystem” in which individuals and organizations can

complete online transactions with 14 ,trusting the identities of the infrastructure that the transaction

runs 15 .'"

Still, the administration’s plan has 16 privacy rights activists. Some applaud the approach; others are

concerned. It seems clear that such an initiative push toward what would 17 be a license” mentality.

The plan has also been greeted with 18 by some experts, who worry that the “voluntary

ecosystem” would still leave much of the Internet 19 .They argue that should be 20 to register

and identify themselves, in drivers must be licensed to drive on public roads.

1.

2.

3.

4.

5.

6.

7.

8.

9.

10.

11.

12.

13.

14.

15.

16.

17.

18.

19.

20.

ss

ation

s

ed

on

vain

d

n

d

stly

cism

able

d

d

s

er

erence

ed

er

t

on

effect

ized

t

ointed

ntally

nce

able

ted

ess

mise

ainment

d

buted

in

return

ng

ence

ted

onally

erence

able

d

ss

al

lent

ed

e

e

red

in

contrast

ing

ce

ally

iasm

ble

Section II Reading Comprehension

11

2010——2019年考研英语二真题及答案

Part A

Directions:

Read the following four texts. Answer the questions after each text by choosing A, B, C or D. Mark your

answers on ANSWER SHEET 1. (40points)

Text 1

Ruth Simmons joined Goldman Sachs’s board as an outside director in January 2000: a year later she

became president of Brown University. For the rest of the decade she apparently managed both roles

without attracting much eroticism. But by the end of 2009 Ms. Simmons was under fire for having sat on

Goldman’s compensation committee; how could she have let those enormous bonus payouts pass

unremarked? By February the next year Ms. Simmons had left the board. The position was just taking up

too much time, she said.

Outside directors are supposed to serve as helpful, yet less biased, advisers on a firm’s board. Having

made their wealth and their reputations elsewhere, they presumably have enough independence to disagree

with the chief executive’s proposals. If the sky, and the share price is falling, outside directors should be

able to give advice based on having weathered their own crises.

The researchers from Ohio University used a database hat covered more than 10,000 firms and more

than 64,000 different directors between 1989 and 2004. Then they simply checked which directors stayed

from one proxy statement to the next. The most likely reason for departing a board was age, so the

researchers concentrated on those “surprise” disappearances by directors under the age of 70. They fount

that after a surprise departure, the probability that the company will subsequently have to restate earnings

increased by nearly 20%. The likelihood of being named in a federal class-action lawsuit also increases,

and the stock is likely to perform worse. The effect tended to be larger for larger firms. Although a

correlation between them leaving and subsequent bad performance at the firm is suggestive, it does not

mean that such directors are always jumping off a sinking ship. Often they “trade up.” Leaving riskier,

smaller firms for larger and more stable firms.

But the researchers believe that outside directors have an easier time of avoiding a blow to their

reputations if they leave a firm before bad news breaks, even if a review of history shows they were on the

board at the time any wrongdoing occurred. Firms who want to keep their outside directors through tough

times may have to create incentives. Otherwise outside directors will follow the example of Ms. Simmons,

once again very popular on campus.

21. According to Paragraph 1, Ms. Simmons was criticized for .

[A]gaining excessive profits

[B]failing to fulfill her duty

[C]refusing to make compromises

[D]leaving the board in tough times

22. We learn from Paragraph 2 that outside directors are supposed to be .

[A]generous investors [B]unbiased executives

[C]share price forecasters [D]independent advisers

12

2010——2019年考研英语二真题及答案

23. According to the researchers from Ohio University after an outside director’s surprise departure, the

firm is likely to .

[A]become more stable [B]report increased earnings

[C]do less well in the stock market [D]perform worse in lawsuits

24. It can be inferred from the last paragraph that outside directors .

[A]may stay for the attractive offers from the firm [B]have often had records of wrongdoings in the firm

[C]are accustomed to stress-free work in the firm [D]will decline incentives from the firm

25. The author’s attitude toward the role of outside directors is .

[A]permissive [B]positive

[C]scornful [D]critical

Text 2

Whatever happened to the death of newspaper? A year ago the end seemed near. The recession

threatened to remove the advertising and readers that had not already fled to the internet. Newspapers like

the San Francisco Chronicle were chronicling their own doom. America’s Federal Trade commission

launched a round of talks about how to save newspapers. Should they become charitable corporations?

Should the state subsidize them ? It will hold another meeting soon. But the discussions now seem out of

date.

In much of the world there is the sign of crisis. German and Brazilian papers have shrugged off the

recession. Even American newspapers, which inhabit the most troubled come of the global industry, have

not only survived but often returned to profit. Not the 20% profit margins that were routine a few years ago,

but profit all the same.

It has not been much fun. Many papers stayed afloat by pushing journalists overboard. The American

Society of News Editors reckons that 13,500 newsroom jobs have gone since 2007. Readers are paying

more for slimmer products. Some papers even had the nerve to refuse delivery to distant suburbs. Yet these

desperate measures have proved the right ones and, sadly for many journalists, they can be pushed further.

Newspapers are becoming more balanced businesses, with a healthier mix of revenues from readers

and advertisers. American papers have long been highly unusual in their reliance on ads. Fully 87% of their

revenues came from advertising in 2008, according to the Organization for Economic Cooperation &

Development (OECD). In Japan the proportion is 35%. Not surprisingly, Japanese newspapers are much

more stable.

The whirlwind that swept through newsrooms harmed everybody, but much of the damage has been

concentrated in areas where newspaper are least distinctive. Car and film reviewers have gone. So have

science and general business reporters. Foreign bureaus have been savagely cut off. Newspapers are less

complete as a result. But completeness is no longer a virtue in the newspaper business.

26. By saying “Newspapers like … their own doom” (Lines 3-4, Para. 1), the author indicates that

newspaper .

[A]neglected the sign of crisis

[B]failed to get state subsidies

13

2010——2019年考研英语二真题及答案

[C]were not charitable corporations

[D]were in a desperate situation

27. Some newspapers refused delivery to distant suburbs probably because .

[A]readers threatened to pay less

[B]newspapers wanted to reduce costs

[C]journalists reported little about these areas

[D]subscribers complained about slimmer products

28. Compared with their American counterparts, Japanese newspapers are much more stable because

they .

[A]have more sources of revenue

[B]have more balanced newsrooms

[C]are less dependent on advertising

[D]are less affected by readership

29. What can be inferred from the last paragraph about the current newspaper business?

[A]Distinctiveness is an essential feature of newspapers.

[B]Completeness is to blame for the failure of newspaper.

[C]Foreign bureaus play a crucial role in the newspaper business.

[D]Readers have lost their interest in car and film reviews.

30. The most appropriate title for this text would be .

[A]American Newspapers: Struggling for Survival

[B]American Newspapers: Gone with the Wind

[C]American Newspapers: A Thriving Business

[D]American Newspapers: A Hopeless Story

Text 3

We tend to think of the decades immediately following World War II as a time of prosperity and

growth, with soldiers returning home by the millions, going off to college on the G. I. Bill and lining up at

the marriage bureaus.

But when it came to their houses, it was a time of common sense and a belief that less could truly be

more. During the Depression and the war, Americans had learned to live with less, and that restraint, in

combination with the postwar confidence in the future, made small, efficient housing positively stylish.

Economic condition was only a stimulus for the trend toward efficient living. The phrase “less is

more” was actually first popularized by a German, the architect Ludwig Mies van der Rohe, who like other

people associated with the Bauhaus, a school of design, emigrated to the United States before World War II

and took up posts at American architecture schools. These designers came to exert enormous influence

on the course of American architecture, but none more so that Mies.

Mies’s signature phrase means that less decoration, properly organized, has more impact that a lot.

Elegance, he believed, did not derive from abundance. Like other modern architects, he employed metal,

glass and laminated wood-materials that we take for granted today buy that in the 1940s symbolized the

14

2010——2019年考研英语二真题及答案

future. Mies’s sophisticated presentation masked the fact that the spaces he designed were small and

efficient, rather than big and often empty.

The apartments in the elegant towers Mies built on Chicago’s Lake Shore Drive, for example, were

smaller-two-bedroom units under 1,000 square feet-than those in their older neighbors along the city’s Gold

Coast. But they were popular because of their airy glass walls, the views they afforded and the elegance of

the buildings’ details and proportions, the architectural equivalent of the abstract art so popular at the time.

The trend toward “less” was not entirely foreign. In the 1930s Frank Lloyd Wright started building

more modest and efficient houses-usually around 1,200 square feet-than the spreading two-story ones he

had designed in the 1890s and the early 20th century.

The “Case Study Houses” commissioned from talented modern architects by California Arts &

Architecture magazine between 1945 and 1962 were yet another homegrown influence on the “less is

more” trend. Aesthetic effect came from the landscape, new materials and forthright detailing. In his Case

Study House, Ralph everyday life - few American families acquired helicopters, though most eventually

got clothes dryers - but his belief that self-sufficiency was both desirable and inevitable was widely shared.

31. The postwar American housing style largely reflected the Americans’ .

[A]prosperity and growth

[B]efficiency and practicality

[C]restraint and confidence

[D]pride and faithfulness

32. Which of the following can be inferred from Paragraph 3 about Bauhaus?

[A]It was founded by Ludwig Mies van der Rohe.

[B]Its designing concept was affected by World War II.

[C]Most American architects used to be associated with it.

[D]It had a great influence upon American architecture.

33. Mies held that elegance of architectural design .

[A]was related to large space

[B]was identified with emptiness

[C]was not reliant on abundant decoration

[D]was not associated with efficiency

34. What is true about the apartments Mies building Chicago’s Lake Shore Drive?

[A]They ignored details and proportions.

[B]They were built with materials popular at that time.

[C]They were more spacious than neighboring buildings.

[D]They shared some characteristics of abstract art.

35. What can we learn about the design of the “Case Study House”?

[A]Mechanical devices were widely used.

[B]Natural scenes were taken into consideration

[C]Details were sacrificed for the overall effect.

15

2010——2019年考研英语二真题及答案

[D]Eco-friendly materials were employed.

Text 4

Will the European Union make it? The question would have sounded strange not long ago. Now even

the project’s greatest cheerleaders talk of a continent facing a “Bermuda triangle” of debt, population

decline and lower growth.

As well as those chronic problems, the EU face an acute crisis in its economic core, the 16 countries

that use the single currency. Markets have lost faith that the euro zone’s economies, weaker or stronger, will

one day converge thanks to the discipline of sharing a single currency, which denies uncompetitive

members the quick fix of devaluation.

Yet the debate about how to save Europe’s single currency from disintegration is stuck. It is stuck

because the euro zone’s dominant powers, France and Germany, agree on the need for greater

harmonization within the euro zone, but disagree about what to harmonies.

Germany thinks the euro must be saved by stricter rules on borrow spending and competitiveness,

barked by quasi-automatic sanctions for governments that do not obey. These might include threats to

freeze EU funds for poorer regions and EU mega-projects and even the suspension of a country’s voting

rights in EU ministerial councils. It insists that economic co-ordination should involve all 27 members of

the EU club, among whom there is a small majority for free-market liberalism and economic rigour; in the

inner core alone, Germany fears, a small majority favour French interference.

A “southern” camp headed by French wants something different: ”European economic government”

within an inner core of euro-zone members. Translated, that means politicians intervening in monetary

policy and a system of redistribution from richer to poorer members, via cheaper borrowing for

governments through common Eurobonds or complete fiscal transfers. Finally, figures close to the France

government have murmured, curo-zone members should agree to some fiscal and social harmonization: e.g.,

curbing competition in corporate-tax rates or labour costs.

It is too soon to write off the EU. It remains the world’s largest trading block. At its best, the European

project is remarkably liberal: built around a single market of 27 rich and poor countries, its internal borders

are far more open to goods, capital and labour than any comparable trading area. It is an ambitious attempt

to blunt the sharpest edges of globalization, and make capitalism benign.

36. The EU is faced with so many problems that .

[A] it has more or less lost faith in markets

[B] even its supporters begin to feel concerned

[C] some of its member countries plan to abandon euro

[D] it intends to deny the possibility of devaluation

37. The debate over the EU’s single currency is stuck because the dominant powers .

[A] are competing for the leading position

[B] are busy handling their own crises

[C] fail to reach an agreement on harmonization

[D] disagree on the steps towards disintegration

16

2010——2019年考研英语二真题及答案

38. To solve the euro problem ,Germany proposed that .

[A] EU funds for poor regions be increased

[B] stricter regulations be imposed

[C] only core members be involved in economic co-ordination

[D] voting rights of the EU members be guaranteed

39. The French proposal of handling the crisis implies that __ __.

[A]poor countries are more likely to get funds

[B]strict monetary policy will be applied to poor countries

[C]loans will be readily available to rich countries

[D]rich countries will basically control Eurobonds

40. Regarding the future of the EU, the author seems to feel __ __.

[A]pessimistic [B]desperate

[C]conceited [D]hopeful

Part B

Directions:

Read the following text and answer the questions by finding information from the right column that

corresponds to each of the marked details given in the left column. There are two extra choices in the right

column. Mark your answer on ANSWER SHEET 1. (10 points)

17

2010——2019年考研英语二真题及答案

Such a move could affect firms such as McDonald’s, which sponsors the youth coaching scheme run

by the Football Association. Fast-food chains should also stop offering “inducements” such as toys, cute

animals and mobile phone credit to lure young customers, Stephenson said.

Professor Dinesh Bhugra, president of the Royal College of Psychiatrists, said: “If children are taught

about the impact that food has on their growth, and that some things can harm, at least information is

available up front.”

He also urged councils to impose “fast-food-free zones” around school and hospitals-areas within

which takeaways cannot open.

A Department of Health spokesperson said: “We need to create a new vision for public health where

18

2010——2019年考研英语二真题及答案

all of society works together to get healthy and live longer. This includes creating a new ‘responsibility

deal’ with business, built on social responsibility, not state regulation. Later this year, we will publish a

white paper setting out exactly how we will achieve this.”

The food industry will be alarmed that such senior doctors back such radical moves, especially the call

to use some of the tough tactics that have been deployed against smoking over the last decade.

Lansley held that

e Stephenson agreed that

Oliver seemed to believe that

[A] “fat taxes” should be imposed on fast-food

producers such as McDonald’s.

[B] the government should ban fast-food outlets

in the neighborhood of schools

[C] “lecturing” was an effective way to

improve school lunches in England.

[D] cigarette-style warnings should be

introduced to children about the dangers of a

poor diet.

Bhugra suggested

that

45.A Department of Health

Spokesperson propsed that

46.Direction:

In this section there is a text in English. Translate it into Chinese, write your translation on ANSWER

SHEET 2. (15points)

Who would have thought that, globally, the IT industry produces about the same volumes of

greenhouse gases as the world’s airlines do-rough 2 percent of all CO2 emissions?

Many everyday tasks take a surprising toll on the environment. A Google search can leak between 0.2

and 7.0 grams of CO2 depending on how many attempts are needed to get the “right” answer. To deliver

results to its users quickly, then, Google has to maintain vast data centres round the world, packed with

powerful computers. While producing large quantities of CO2, these computers emit a great deal of heat, so

the centres need to be well air-conditioned, which uses even more energy.

However, Google and other big tech providers monitor their efficiency closely and make

improvements. Monitoring is the first step on the road to reduction, but there is much to be done, and not

just by big companies.

[E] the producers of crisps and candies could

contribute significantly to the Change4Life

campaign.

[F] parents should set good examples for their

children by keeping a healthy diet at home.

[G] the government should strengthen the sense

of responsibility among businesses.

Section IV Writing

Part A

47 Directions:

19

2010——2019年考研英语二真题及答案

Suppose your cousin Li Ming has just been admitted to a university. Write him/her a letter to

1) congratulate him/her, and

2) give him/her suggestions on how to get prepared for university life.

You should write about 100 words on ANSWER SHEET 2.

Do not sign your own name at the end of the letter. Use “Zhang Wei” instead.

Do not write the address. (10 points)

Part B

48Directions:

Write a short essay baesd on the following your writing,you should:

1)interpret the chart and

2)give your comments

you should write at least 150 wrods

write your essay on answer sheet 2(15points)

2011英语二参考答案

1.A 2.C 3.B 4.D 5.D 6.B 7.A 8.C 9.C 10.B

11.D 12.B 13.A 14.C 15.A 16.A 17.D 18.A 19.C 20.D

21-25 BDCAD 26-30 DBCAA 31-35 CDCDB 36-40 BCBAD

41.E 42.D 43.C 44.B 45.G

参考答案

从全球范围来看,有谁会想到IT 行业释放的温室气体与全球航空公司产生的一样多呢?它大约占总二氧化碳总排量的2%。

许多日常工作对环境造成了令人震惊的破坏。根据每次你搜索并得到正确答案的尝试次数,谷歌会排放0.2至7克的二氧化碳。为了迅速将结果传递给用户,谷歌在全球设置了大量充斥着能量巨大的电脑的数据中心。这些电脑在排放大量二氧化碳的同时,也产生大量的能量。因此,这些数据中心需要良好的空调降温,这又会同时产生大量的能量。

20

2010——2019年考研英语二真题及答案

然而,谷歌和其他技术提供商严密检测他们的效果并不断进行改进。监控是减排的第一步,但这仍任重道远,且不仅只由大公司来承担。

小作文参考答案

Dear friend,

I am writing to congratulate you on your being successfully admitted to Harvard University, which

enjoys an international reputation for its academic excellence and give you some suggestions as to how to

make preparation for the coming college life.

In order for you to adapt yourself to the university life, you are advised to get prepared physically and

intellectually. First and foremost, you need to build a strong body for the future academic pursuit, so you

can take some exercises during the breaks. Secondly, since the study in university is more demanding than

in your secondary school, you are highly suggested to find some introductory books from the library so as

to have a good idea of the specialty you are going to take in your college life. Given your sound ability, you

are sure to have a successful college life.

Congratulate you again and wish you a fruitful college life.

Sincerely yours,

Zhang Wei

Dear Ming,

Congratulations! I am glad to hear that you have been admitted by MIT. Your efforts and commitment

have been paid off. You are the honor of our family.

Here come some my own advices of being a pre college student. First and foremost, you need to

improve your communication because you will meet different people with different personalities in campus.

Moreover, reading some reference books will help you to accumulate more knowledge and terms, which

boost your competitiveness in campus.

Once again congratulate for your achievement!

Yours sincerely,

Zhang Wei

大作文参考答案

As is shown in the bar chart above, dramatic changes have taken place in the autos market shares

within two years (from 2008 to 2009). The most obvious change was the market share of national brand,

which had increased nearly by 10%, while Japan’s autos market share decreased roughly by 10%. The

percentage of the US autos remained stable between 2008 and 2009.

There are numerous reasons accounting for the phenomenon and I would like to explore a few of the

most important ones here. Above all, as the development of technique and knowledge in native companies,

a growing number of autos corporation developed many quality autos. Therefore, the national people

changed the attitude to the native brands and acknowledge them. What’ more, an overwhelming majority of

people have been affected by the country patriotism ideology, partly owing to some actions of Japan

triggering the emotion of people. Finally, Toyota brake error accidents significantly affects Japanese autos’

reputations and images. Safety concerns drove customers away from Japanese products. Additionally, Fuel

21

2010——2019年考研英语二真题及答案

price drove consumers away from those American petrol digging and luxury autos. So it is not difficult to

observe their steady performance.

Based on what has been discussed above, we may reasonably conclude that the tendency described in

graphic will continue for quite a long time. Hopefully, government could offer more friendly policies to

China autos manufacturers to encourage quality improvement and technology innovation.

2012年研究生入学考试真题及解析

Section 1 Use of Eninglish

Directions :

Millions of Americans and foreigners see as a mindless war toy ,the symbol of American

military adventurism, but that’s not how it used to be .To the men and women who 1 )in World War II and

the people they liberated ,the the 2) man grown into hero ,the pool farm kid torn away from his

home ,the guy who 3) all the burdens of battle ,who slept in cold foxholes,who went without the 4) of food

and shelter ,who stuck it out and drove back the Nazi reign of murder .this was not a volunteer soldier ,not

someone well paid ,5) an average guy ,up 6 )the best trained ,best equipped ,fiercest ,most brutal enemies

seen in centuries.

His name is not . is just a military abbreviation 7) Government Issue ,and it was on all of the

article 8) to soldiers .And Joe? A common name for a guy who never 9) it to the top .Joe Blow ,Joe

Magrac …a working class United States has 10) had a president or vicepresident or secretary of

state Joe.

GI .joe had a (11)career fighting German ,Japanese , and Korean troops . He appers as a

character ,or a (12 ) of american personalities, in the 1945 movie The Story of GI. Joe, based on the last

days of war correspondent Ernie Pyle. Some of the soldiers Pyle(13)portrayde themselves in the film. Pyle

was famous for covering the (14)side of the warl, writing about the dirt-snow –and-mud soldiers, not how

many miles were(15)or what towns were captured or liberated, His reports(16)the “willie” cartoons of

famed Stars and Stripes artist Bill Maulden. Both men(17)the dirt and exhaustion of war, the (18)of

civilization that the soldiers shared with each other and the civilians: coffee, tobacco, whiskey, shelter,

sleep. (19)Egypt, France, and a dozen more countries, G.I. Joe was any American soldier,(20)the most

important person in their lives.

1.[A] performed

2.[A] actual

3.[A]bore

5.[A]and

6.[A]for

4.[A]necessities

[B]served

[B]cased

[B]nor

[C]rebelled

[D]betrayed

[D]normal

[D]loaded

[D]propertoes

[D]hence

[D]against

[D]claiming

22

[B]common [C]special

[C]removed

[C]but

[B]facilitice [C]commodities

[B]into [C] form

7.[A]meaning [B]implying [C]symbolizing

2010——2019年考研英语二真题及答案

8.[A]handed out

9.[A]pushed

10.[A]ever

11.[A]disguised

12.[A]company

13.[A]employed

14.[A]ethical

15.[A]ruined

16.[A]paralleled

17.[A]neglected

18.[A]stages

19.[A]With

[B]turn over [C]brought back

[B]got [C]made

[C]either

[B]never

[D]passed down

[D]managed

[D]neither

[D]distinguished

[D]colony

[D]questioned

[D]human

[D]gained

[D]contradicted

[D]admired

[D]advancea

[D]Beyond

[B]disturbed [C]disputed

[B]collection [C]community

[B]appointed [C]interviewed

[B]military [C]political

[B]commuted [C]patrolled

[B]counteracted

[B]avoided

[B]illusions

[B]To

[C]duplicated

[C]emphasized

[C]fragments

[C]Among

20.[A]on the contrary [B] by this means [C]from the outset [D]at that point

Section II Resdiong Comprehension

Part A

Directions:

Read the following four texts. answer the question after each text by choosing A,B,C or D. Mark your

answers on ANSWER SHEET 1.(40 points)

Text 1

Homework has never been terribly popular with students and even many parents, but in recent years it

has been particularly scorned. School districts across the country, most recently Los Angeles Unified, are

revising their thinking on his educational ritual. Unfortunately, L.A. Unified has produced an inflexible

policy which mandates that with the exception of some advanced courses, homework may no longer count

for more than 10% of a student’s academic grade.

This rule is meant to address the difficulty that students from impoverished or chaotic homes might

have in completing their homework. But the policy is unclear and contradictory. Certainly, no homework

should be assigned that students cannot do without expensive equipment. But if the district is essentially

giving a pass to students who do not do their homework because of complicated family lives, it is going

riskily close to the implication that standards need to be lowered for poor children.

District administrators say that homework will still be a pat of schooling: teachers are allowed to

assign as much of it as they want. But with homework counting for no more than 10% of their grades,

students can easily skip half their homework and see vey little difference on their report cards. Some

students might do well on state tests without completing their homework, but what about the students who

performed well on the tests and did their homework? It is quite possible that the homework helped. Yet

rather than empowering teachers to find what works best for their students, the policy imposes a flat,

across-the-board rule.

23

2010——2019年考研英语二真题及答案

At the same time, the policy addresses none of the truly thorny questions about homework. If the

district finds homework to be unimportant to its students’ academic achievement, it should move to reduce

or eliminate the assignments, not make them count for almost nothing. Conversely, if homework does

nothing to ensure that the homework students are not assigning more than they are willing to review and

correct.

The homework rules should be put on hold while the school board, which is responsible for setting

educational policy, looks into the matter and conducts public hearings. It is not too late for L.A. Unified to

do homework right.

is implied in paragraph 1 that nowadays homework_____.

[A] is receiving more criticism

[B]is no longer an educational ritual

[C]is not required for advanced courses

[D]is gaining more preferences

d has made the rule about homework mainly because poor students_____.

[A]tend to have moderate expectations for their education

[B]have asked for a different educational standard

[C]may have problems finishing their homework

[D]have voiced their complaints about homework

ing to Paragraph 3,one problem with the policy is that it may____.

[A]discourage students from doing homework

[B]result in students' indifference to their report cards

[C]undermine the authority of state tests

[D]restrict teachers' power in education

24. As mentioned in Paragraph 4, a key question unanswered about homework is whether______.

[A] it should be eliminated [B]it counts much in schooling

[C]it places extra burdens on teachers [D]it is important for grades

25.A suitable title for this text could be______.

[A]Wrong Interpretation of an Educational Policy

[B]A Welcomed Policy for Poor Students

[C]Thorny Questions about Homework

[D]A Faulty Approach to Homework

Text2

Pretty in pink: adult women do not rememer being so obsessed with the colour, yet it is pervasive in

our young girls’ lives. Tt is not that pink is intrinsically bad, but it is such a tiny slice of the rainbow and,

though it may celebrate girlhood in one way, it also repeatedly and firmly fuses girls’ identity to appearance.

Then it presents that connection, even among two-year-olds, between girls as not only innocent but as

evidence of innocence. Looking around, I despaired at the singular lack of imagination about girls’ lives

and interests.

24

2010——2019年考研英语二真题及答案

Girls’ attraction to pink may seem unavoidable, somehow encoded in their DNA, but according to Jo

Paoletti, an associate professor of American Studies, it is not. Children were not colour-coded at all until

the early 20th century: in the era before domestic washing machines all babies wore white as a practical

matter, since the only way of getting clothes clean was to boil them. What’s more, both boys and girls wore

what were thought of as gender-neutral nursery colours were introduced, pink was actually

considered the more masculine colour, a pastel version of red, which was associated with strength. Blue,

with its intimations of the Virgin Mary, constancy and faithfulness, symbolised femininity. It was not until

the mid-1980s, when amplifying age and sex differences became a dominant children’s marketing strategy,

that pink fully came into its own, when it began to seem inherently attractive to girls, part of what defined

them as female, at least for the first few critical years.

I had not realised how profoundly marketing trends dictated our perception of what is natural to kins,

including our core beliefs about their psychological development. Take the toddler. I assumed that phase

was something experts developed after years of research into children’s behaviour: wrong. Turns out,

acdording to Daniel Cook, a historian of childhood consumerism, it was popularised as a marketing trick by

clothing manufacrurers in the 1930s.

Trade publications counselled department stores that, in order to increase sales, they should create a

“third stepping stone” between infant wear and older kids’ clothes. Tt was only after “toddler”became a

common shoppers’ term that it evolved into a broadly accepted developmental stage. Splitting kids, or

adults,into ever-tinier categories has proved a sure-fire way to boost profits. And one of the easiest ways to

segment a market is to magnify gender differences – or invent them where they did not previously exist.

saying "he rainbow"(Line 3, Para.1),the author means pink______.

[A]should not be the sole representation of girlhood

[B]should not be associated with girls' innocence

[C]cannot explain girls' lack of imagination

[D]cannot influence girls' lives and interests

ing to Paragraph 2, which of the following is true of colours?

[A]Colours are encoded in girls' DNA.

[B]Blue used to be regarded as the colour for girls.

[C]Pink used to be a neutral colour in symbolising genders.

[D]White is prefered by babies.

author suggests that our perception of children's psychological development was much influenced

by_____.

[A]the marketing of products for children

[B]the observation of children's nature

[C]researches into children's behavior

[D]studies of childhood consumption

may learn from Paragraph 4 that department stores were advised to_____.

[A]focus on infant wear and older kids' clothes

25

2010——2019年考研英语二真题及答案

[B]attach equal importance to different genders

[C]classify consumers into smaller groups

[D]create some common shoppers' terms

can be concluded that girls' attraction to pink seems to be____.

[A] clearly explained by their inborn tendency

[B]fully understood by clothing manufacturers

[C] mainly imposed by profit-driven businessmen

[D]well interpreted by psychological experts

Text 3

In 2010. a federal judge shook America's biotech industry to its core. Companies had won patents for

isolated DNA for decades-by 2005 some 20% of human genes were parented. But in March 2010 a judge

ruled that genes were unpatentable. Executives were violently agitated. The Biotechnology Industry

Organisation (BIO), a trade group, assured members that this was just a “preliminary step” in a longer

battle.

On July 29th they were relieved, at least temporarily. A federal appeals court overturned the prior

decision, ruling that Myriad Genetics could indeed holb patents to two genss that help forecast a woman's

risk of breast cancer. The chief executive of Myriad, a company in Utah,said the ruling was a blessing to

firms and patients alike.

But as companies continue their attempts at personalised medicine, the courts will remain rather busy.

The Myriad case itself is probably not over Critics make three main arguments against gene patents: a

gene is a product of nature, so it may not be patented; gene patents suppress innovation rather than

reward it; and patents' monopolies restrict access to genetic tests such as Myriad's. A growing number

seem to year a federal task-force urged reform for patents related to genetic tests. In October the

Department of Justice filed a brief in the Myriad case, arguing that an isolated DNA molecule “is no less a

product than are cotton fibres that have been separated from cotton seeds. ”

Despite the appeals court's decision, big questions remain unanswered. For example, it is unclear

whether the sequencing of a whole genome violates the patents of indivi dual genes within it. The case may

yet reach the Supreme Court.

AS the industry advances ,however,other suits may have an even greater ies are

unlikely to file many more patents for human DNA molecules-most are already patented or in the public

domain .firms are now studying how genes intcract,looking for correlations that might be used to determine

the causes of disease or predict a drug’s efficacy,companies are eager to win patents for ‘connecting the

dits’,expaains hans sauer,alawyer for the BIO.

Their success may be determined by a suit related to this issue, brought by the Mayo Clinic, which the

Supreme Court will hear in its next term. The BIO rtcently held a convention which included seddions to

coach lawyers on the shifting landscape for patents. Each meeting was packed.

canbe learned from paragraph I that the biotech companies would like-----

executives to be active to rule out gene patenting

26

2010——2019年考研英语二真题及答案

to be patcntablc BIO to issue a warning

who are against gene patents believe that----

c tests are not reliable

man-made products are patentable

s on genes depend much on innovatiaon

should restrict access to gene tic tests

ing to hans sauer ,companies are eager to win patents for----

ishing disease comelations ering gene interactions

g pictures of genes fying human DNA

34.By saying “each meeting was packed”(line4,para6)the author means that -----

supreme court was authoritative

BIO was a powerful organization

patenting was a great concern

s were keen to attend conventiongs

lly speaking ,the author’s attitude toward gene patenting is----

al tive

ul ive

Text 4

The great recession may be over, but this era of high joblessness is probably beginning. Before it ends,

it will likely change the life course and character of a generation of young adults. And ultimately, it is

likely to reshape our politics,our culture, and the character of our society for years.

No one tries harder than the jobless to find silver linings in this national economic disaster. Many said

that unemployment, while extremely painful, had improved them in some ways; they had become less

materialistic and more financially prudent; they were more aware of the struggles of others. In limited

respects, perhaps the recession will leave society better off. At the very least, it has awoken us from our

national fever dream of easy riches and bigger houses, and put a necessary end to an era of reckless

personal spending.

But for the most part, these benefits seem thin, uncertain, and far off. In The Moral Consequences of

Economic Growth, the economic historian Benjamin Friedman argues that both inside and outside the

U.S. ,lengthy periods of economic stagnation or decline have almost always left society more mean-spirited

and less inclusive, and have usually stopped or reversed the advance of rights and freedoms.

Anti-immigrant sentiment typically increases, as does conflict between races and classes.

Income inequality usually falls during a recession, but it has not shrunk in this one,. Indeed, this period

of economic weakness may reinforce class divides, and decrease opportunities to cross them--- especially

for young people. The research of Till Von Wachter, the economist in Columbia University, suggests that

not all people graduating into a recession see their life chances dimmed: those with degrees from elite

universities catch up fairly quickly to where they otherwise would have been if they had graduated in better

times; it is the masses beneath them that are left behind.

27

2010——2019年考研英语二真题及答案

In the internet age, it is particularly easy to see the resentment that has always been hidden winthin

American society. More difficult, in the moment , is discerning precisely how these lean times are affecting

society’s character. In many respects, the U.S. was more socially tolerant entering this resession than at any

time in its history, and a variety of national polls on social conflict since then have shown mixed results. We

will have to wait and see exactly how these hard times will reshape our social fabric. But they certainly it,

and all the more so the longer they extend.

saying “to find silver linings”(Line 1,Para.2)the author suggest that the jobless try to___.

[A]seek subsidies from the govemment

[B]explore reasons for the unermployment

[C]make profits from the troubled economy

[D]look on the bright side of the recession

ing to Paragraph 2,the recession has made people_____.

[A]realize the national dream [B]struggle against each other

[C]challenge their lifestyle [D]reconsider their lifestyle

in Friedman believe that economic recessions may_____.

[A]impose a heavier burden on immigrants

[B]bring out more evils of human nature

[C]Promote the advance of rights and freedoms

[D]ease conflicts between races and classes

research of Till Von Wachther suggests that in recession graduates from elite universities tend to

_____.

[A]lag behind the others due to decreased opportunities

[B]catch up quickly with experienced employees

[C]see their life chances as dimmed as the others’

[D]recover more quickly than the others

author thinks that the influence of hard times on society is____.

[A]certain [B]positive

[C]trivial [D]destructive

Part B

Directions:

Read the following text and answer the questions by finding information from the left column that

corresponds to each of the marked details given in the right column. There are two extra choices in the right

column. Mark your answers on ANSWER SHEERT 1.(10 points)

“Universal history, the history of what man has accomplished in this world, is at bottom the

History of the Great Men who have worked here,” wrote the Victorian sage Thomas Carlyle. Well, not any

more it is not.

Suddenly, Britain looks to have fallen out with its favourite historical form. This could be no more

28

2010——2019年考研英语二真题及答案

than a passing literary craze, but it also points to a broader truth about how we now approach the past: less

concerned with learning from forefathers and more interested in feeling their pain. Today, we want empathy,

not inspiration.

From the earliest days of the Renaissance, the writing of history meant recounting the exemplary lives

of great men. In 1337, Petrarch began work on his rambling writing De Viris Illustribus – On Famous Men,

highlighting the virtus (or virtue) of classical heroes. Petrarch celebrated their greatness in conquering

fortune and rising to the top. This was the biographical tradition which Niccolo Machiavelli turned on its

head. In The Prince, the championed cunning, ruthlessness, and boldness, rather than virtue, mercy and

justice, as the skills of successful leaders.

Over time, the attributes of greatness shifted. The Romantics commemorated the leading painters and

authors of their day, stressing the uniqueness of the artist's personal experience rather than public glory. By

contrast, the Victorian author Samual Smiles wrote Self-Help as a catalogue of the worthy lives of

engineers , industrialists and explores . "The valuable examples which they furnish of the power of

self-help, if patient purpose, resolute working and steadfast integrity, issuing in the formulation of truly

noble and many character, exhibit,"wrote Smiles."what it is in the power of each to accomplish for

himself"His biographies of James Walt, Richard Arkwright and Josiah Wedgwood were held up as beacons

to guide the working man through his difficult life.

This was all a bit bourgeois for Thomas Carlyle, who focused his biographies on the truly heroic lives

of Martin Luther, Oliver Cromwell and Napoleon Bonaparte. These epochal figures represented lives hard

to imitate, but to be acknowledged as possessing higher authority than mere mortals.

Communist Manifesto. For them, history did nothing, it possessed no immense wealth nor waged

battles:“It is man, real, living man who does all that.” And history should be the story of the masses and

their record of struggle. As such, it needed to appreciate the economic realities, the social contexts and

power relations in which each epoch stood. For:“Men make their own history, but they do not make it just

as they please; they do not make it under circumstances chosen by themselves, but under circumstances

directly found, given and transmitted from the past.”

This was the tradition which revolutionized our appreciation of the past. In place of Thomas Carlyle,

Britain nurtured Christopher Hill, EP Thompson and Eric Hobsbawm. History from below stood alongside

biographies of great men. Whole new realms of understanding — from gender to race to cultural studies —

were opened up as scholars unpicked the multiplicity of lost societies. And it transformed public history too:

downstairs became just as fascinating as upstairs.

41. Petrarch

42. Niccolo Machiavellli

43. Samuel Smiles

[A] emphasized the virtue of classical heroes.

[B] highlighted the public glory of the leading

artists.

[C] focused on epochal figures whose lives

were hard to imitate

[D] opened up new realms of understanding the

29

2010——2019年考研英语二真题及答案

great men in history.

44. Thomas Carlyle

45. Marx and Engels

[E] held that history should be the story of the

masses and their record of struggle.

[F] dismissed virtue as unnecessary for

successful leaders.

[G] depicted the worthy lives of engineer

industrialists and explorers.

Section III Translation

ions:

Translate the following text from English into your translation on ANSWER SHEET2.(15

points)

When people in developing countries worry about migration,they are usually concerned at the

prospect of ther best and brightest departure to Silicon Valley or to hospitals and universities in the

developed world ,These are the kind of workers that countries like Britian ,Canada and Australia try to

attract by using immigration rules that privilege college graduates .

Lots of studies have found that well-educated people from developing countries are particularly

likely to emigrate .A big survey of Indian households in 2004 found that nearly 40%of emigrants had

more than a high-school education,compared with around 3.3%of all Indians over the age of

"brain drain "has long bothered policymakers in poor countries ,They fear that it hurts their

economies ,depriving them of much-needed skilled workers who could have taught at their

universities ,worked in their hospitals and come up with clever new products for their factories to make .

Section IV Writing

Part A

ions

Suppose you have found something wrong with the electronic dictionary that you bought from an

onlin store the other day ,Write an email to the customer service center to

1)make a complaint and

2)demand a prompt solution

You should write about 100words on ANSERE SHEET 2

Do not sign your own name at the end of the letter ,Use "zhang wei "instead .

Part B

ions

write an essay based on the following table .In your writing you should

1)describe the table ,and

2)give your comments

30

2010——2019年考研英语二真题及答案

You should write at least 150 words(15points)

某公司员工工作满意度调查

年龄 -------满意度

小于等于40岁

41-50岁

大于50岁

2012考研英语二参考答案

满意

16.7%

0.0%

40.0

不清楚

50.0%

36.0%

50.0%

不满意

33.3%

64.0%

10.0%

1.B 2.B 3.A 4.A 5.C 6.B 7.C 8.A 9.D 10.B 11.D 12.B 13.C 14.D 15.B

16.A 17.C 18.B 19.B 20.D 21. A 22.C 23.A 24.B 25.D 26.A 27.B 28.A 29.C 30.C

31.C 32.B 33.A 34.D 35.D 36.D 37.D 38.B 39.D 40.A

41-45:AFGCE

小作文范文:

Dear Sir or Madame,

As one of the regular customers of your online store, I am writing this letter to express my complaint

against the flaws in your product—an electronic dictionary I bought in your shop the other day.

The dictionary is supposed to be a favorable tool for my study. Unfortunately, I found that there are

several problems. To begin with, when I opened it, I detected that the appearance of it had been scratched.

Secondly, I did not find the battery promised in the advertisement posted on the homepage of your shop,

which makes me feel that you have not kept your promise. What is worse, some of the keys on the

keyboard do not work.

I strongly request that a satisfactory explanation be given and effective measures should be taken to

improve your service and the quality of your products. You can either send a new one to me or refund me

my money in full.

I am looking forward to your reply at your earliest convenience.

Sincerely yours,

Zhang Wei

大作文范文:

The table above revealed an overall picture of employment satisfaction. Based upon the data of the

table, most people under 40 are unclear or dissatisfied with their job, and 64% of those between 40 to 50,

are not satisfied and no one feel satisfied at all. For people over 50, the degree of satisfaction largely

exceeds the other groups, amounting to 40%.

Such difference may be rooted in the following reasons. First, middle-aged people face more pressure

to support the family, both the children and the senior, so that they neglect to enjoy in work. Second, the

senior citizens has developed a lot in personality, so they are more prone to see the optimistic aspects of the

work. Last, the conclusion that the current society patterns pose more challenges to the middle aged group

under 50.

To sum up, the senior citizens enjoys more content than the young and middle-aged people under 50.

31

2010——2019年考研英语二真题及答案

2013考研英语(二)真题及答案解析

Section I Use of English

Directions:

Read the following text. Choose the best word(s) for each numbered blank and mark A, B, C or D on

ANSWER SHEET 1. (10 points)

Given the advantages of electronic money, you might think that we would move quickly to the

cashless society in which all payments are made electronically. 1 a true cashless society is probably not

around the corner. Indeed, predictions have been 2 for two decades but have not yet come to fruition. For

example, Business Week predicted in 1975 that electronic means of payment would soon "revolutionize the

very 3 of money itself," only to 4 itself several years later. Why has the movement to a cashless society

been so 5 in coming?

Although electronic means of payment may be more efficient than a payments system based on paper,

several factors work 6 the disappearance of the paper system. First, it is very 7 to set up the computer, card

reader, and telecornmunications networks necessary to make electronic money the 8 form of payment

Second, paper checks have the advantage that they 9 receipts, something thai many consumers are

unwilling to 10 . Third, the use of paper checks gives consumers several days of "float" - it takes several

days 11 a check is cashed and funds are 12 from the issuer's account, which means that the writer of the

check can cam interest on the funds in the meantime. 13 electronic payments arc immediate, they eliminate

the float for the consumer.

Fourth, electronic means of payment may 14 security and privacy concerns. We often hear media

reports that an unauthorized hacker has been able to access a computer database and to alter information 15

there. The fact that this is not an 16 occurrence means that dishonest persons might be able to access bank

accounts in electronic payments systems and 17 from someone else's accounts. The 18 of this type of fraud

is no easy task, and a new field of computer science is developing to 19 security issues. A further concern is

that the use of e lectronic means of payment leaves an electronic 20 that contains a large amount of

personal data. There are concerns that government, employers, and marketers might be able to access these

data, thereby violating our privacy.

1. [A] However [B] Moreover [C] Therefore [D] Otherwise

2. [A] off [B] back [C] over [D] around

3. [A] power [B] concept [C] history [D] role

4. [A] reward [B] resist [C] resume [D] reverse

5. [A] silent [B] sudden [C] slow [D] steady

6. [A] for [B] against [C] with [D] on

7. [A] imaginative [B] expensive [C] sensitive [D] productive

32

2010——2019年考研英语二真题及答案

8. [A] similar [B] original [C] temporary [D] dominant

9. [A] collect [B] provide [C] copy [D] print

10. [A] give up [B] take over [C] bring back [D] pass down

11. [A] before [B] after [C] since [D] when

12. [A] kept [B] borrowed [C] released [D] withdrawn

13. [A] Unless [B] Until [C] Because [D] Though

14. [A] hide [B] express [C] raise [D]ease

15. [A] analyzed [B] shared [C] stored [D] displayed

16. [A] unsafe [B] unnatural [C] uncommon [D] unclear

17. [A] steal [B] choose [C] benefit [D] return

18. [A] consideration [B] prevention [C] manipulation [D] justification

19. [A] cope with [B] fight against [C] adapt to [D] call for

20. [A] chunk [B] chip [C] path [D] trail

Section II Reading Comprehension

Part A

Directions:

Read the following four texts. Answer the questions below each text by choosing A, B, C or D. Mark

your answers on ANSWER SHEET 1. (40 points)

Text 1

In an essay entitled “Making It in America”, the author Adam Davidson relates a joke from cotton

about just how much a modern textile mill has been automated: The average mill only two employees

today,” a man and a dog. The man is there to feed the dog is there to keep the man away from the

machines.”

Davidson’s article is one of a number of pieces that have recently appeared making the point that the

reason we have such stubbornly high unemployment and declining middle-class incomes today is also

because of the advances in both globalization and the information technology revolution, which are more

rapidly than ever replacing labor with machines or foreign worker.

In the past, workers with average skills, doing an average job,could earn an average

lifestyle ,But ,today ,average is officially over. Being average just won’t earn you what it used to. It can’t

when so many more employers have so much more access to so much more above average cheap foreign

labor, cheap robotics, cheap software, cheap automation and cheap genius. Therefore, everyone needs to

find their extra-their unique value contribution that makes them stand out in whatever is their field of

employment.

Yes, new technology has been eating jobs forever, and always will. But there’s been an acceleration.

As Davidson notes,” In the 10 years ending in 2009, [U.S.] factories shed workers so fast that they erased

almost all the gains of the previous 70 years; roughly one out of every three manufacturing jobs-about 6

million in total -disappeared.

There will always be changed-new jobs, new products, new services. But the one thing we know for

33

2010——2019年考研英语二真题及答案

sure is that with each advance in globalization and the I.T. revolution, the best jobs will require workers to

have more and better education to make themselves above average.

In a world where average is officially over, there are many things we need to do to support

employment, but nothing would be more important than passing some kind of for the 21st century

that ensures that every American has access to poet-high school education.

21. The joke in Paragraph 1 is used to illustrate_______

[A] the impact of technological advances

[B] the alleviation of job pressure

[C] the shrinkage of textile mills

[D] the decline of middle-class incomes

22. According to Paragraph 3, to be a successful employee, one has to______

[A] work on cheap software

[B] ask for a moderate salary

[C] adopt an average lifestyle

[D] contribute something unique

23. The quotation in Paragraph 4 explains that ______

[A] gains of technology have been erased

[B] job opportunities are disappearing at a high speed

[C] factories are making much less money than before

[D] new jobs and services have been offered

24. According to the author, to reduce unemployment, the most important is_____

[A] to accelerate the I.T. revolution

[B] to ensure more education for people

[C] ro advance economic globalization

[D] to pass more bills in the 21st century

25. Which of the following would be the most appropriate title for the text?

[A] New Law Takes Effect

[B] Technology Goes Cheap

[C] Average Is Over

[D] Recession Is Bad

Text 2

A century ago, the immigrants from across the Atlantic inclued settlers and sojourners. Along with the

many folks looking to make a permanent home in the United States came those who had no intention to

stay, and 7millin people arrived while about 2 million departed. About a quarter of all Italian immigrants,

for exanmle, eventually returned to Italy for good. They even had an affectionate nickname, “uccelli di

passaggio,” birds of passage.

Today, we are much more rigid about immigrants. We divide nemcomers into two categories: legal or

illegal, good or bad. We hail them as Americans in the making, or our broken immigrantion system and the

34

2010——2019年考研英语二真题及答案

long political paralysis over how to fix it. We don’t need more categories, but we need to change the way

we think about categories. We need to look beyond strick definitions of legal and illegal. To start, we can

recognize the new birds of passage, those living and thriving in the gray areas. We might then begin to

solve our immigration challenges.

Crop pickers, violinists, construction workers, entrepreneurs, engineers, home health-care aides and

physicists are among today’s birds of passage. They are energetic participants in a global economy driven

by the flow of work, money and ideas .They prefer to come and go as opportunity calls them , They can

manage to have a job in one place and a family in another.

With or without permission, they straddle laws, jurisdictions and identities with ease. We need them to

imagine the United States as a place where they can be productive for a while without committing

themselves to staying forever. We need them to feel that home can be both here and there and that they can

belong to two nations honorably.

Accommodating this new world of people in motion will require new attitudes on both sides of the

immigration battle .Looking beyond the culture war logic of right or wrong means opening up the middle

ground and understanding that managing immigration today requires multiple paths and multiple outcomes.

Including some that are not easy to accomplish legally in the existing system.

26 “Birds of passage” refers to those who____

[A] immigrate across the Atlantic.

[B] leave their home countries for good.

[C] stay in a foregin temporaily.

[D] find permanent jobs overseas.

27 It is implied in paragraph 2 that the current immigration stystem in the US____

[A] needs new immigrant categories.

[B] has loosened control over immigrants.

[C] should be adopted to meet challenges.

[D] has been fixeed via political means.

28 According to the author, today’s birds of passage want___

[A] fiancial incentives.

[B] a global recognition.

[C] opportunities to get regular jobs.

[D] the freedom to stay and leave.

29 The author suggests that the birds of passage today should be treated __

[A] as faithful partners.

[B] with economic favors.

[C] with regal tolerance.

[D] as mighty rivals.

30 选出最适合文章的标题

[A] come and go: big mistake.

35

2010——2019年考研英语二真题及答案

[B] living and thriving : great risk.

[C] with or without : great risk.

[D] legal or illegal: big mistake.

Text 3

Scientists have found that although we are prone to snap overreactions, if we take a moment and think

about how we are likely to react, we can reduce or even eliminate the negative effects of our quick,

hard-wired responses.

Snap decisions can be important defense mechanisms; if we are judging whether someone is

dangerous, our brains and bodies are hard-wired to react very quickly, within milliseconds. But we need

more time to assess other factors. To accurately tell whether someone is sociable, studies show, we need at

least a minute, preferably five. It takes a while to judge complex aspects of personality, like neuroticism or

open-mindedness.

But snap decisions in reaction to rapid stimuli aren’t exclusive to the interpersonal realm.

Psychologists at the University of Toronto found that viewing a fast-food logo for just a few milliseconds

primes us to read 20 percent faster, even though reading has little to do with eating. We unconsciously

associate fast food with speed and impatience and carry those impulses into whatever else we’re doing,

Subjects exposed to fast-food flashes also tend to think a musical piece lasts too long.

Yet we can reverse such influences. If we know we will overreact to consumer products or housing

options when we see a happy face (one reason good sales representatives and real estate agents are always

smiling), we can take a moment before buying. If we know female job screeners are more likely to reject

attractive female applicants, we can help screeners understand their biases-or hire outside screeners.

John Gottman, the marriage expert, explains that we quickly “thin slice” information reliably only

after we ground such snap reactions in “thick sliced” long-term study. When Dr. Gottman really wants to

assess whether a couple will stay together, he invites them to his island retreat for a muck longer evaluation;

two days, not two seconds.

Our ability to mute our hard-wired reactions by pausing is what differentiates us from animals: doge

can think about the future only intermittently or for a few minutes. But historically we have spent about 12

percent of our days contemplating the longer term. Although technology might change the way we react, it

hasn’t changed our nature. We still have the imaginative capacity to rise above temptation and reverse the

high-speed trend.

31. The time needed in making decisions may____.

[A] vary according to the urgency of the situation

[B] prove the complexity of our brain reaction

[C] depend on the importance of the assessment

[D] predetermine the accuracy of our judgment

32. Our reaction to a fast-food logo shows that snao decisions____.

[A] can be associative

[B] are not unconscious

36

2010——2019年考研英语二真题及答案

[C] can be dangerous

[D] are not impulsive

33. Toreverse the negative influences of snap decisions,we should____.

[A] trust our first impression

[B] do as people usually do

[C] think before we act

[D] ask for expert advice

34. John Gottman says that reliable snap reaction are based on____.

[A] critical assessment

[B]‘‘thin sliced ’’study

[C] sensible explanation

[D] adequate information

35. The author’s attitude toward reversing the high-speed trend is____.

[A] tolerant

[B] uncertain

[C] optimistic

[D] doubtful

Text 4

Europe is not a gender-equality particular, the corporate workplace will never be completely

family—friendly until women are part of senior management decisions,and Europe,s top

corporate-governance positions remain overwhelmingly male .indeed,women hold only 14 percent of

positions on Europe corporate boards.

The Europe Union is now considering legislation to compel corporate boards to maintain a certain

proportion of women-up to 60 proposed mandate was born of frustration. Last year, Europe

Commission Vice President Viviane Reding issued a call to voluntary action. Reding invited corporations

to sign up for gender balance goal of 40 percent female board membership. But her appeal was considered

a failure: only 24 companies took it up.

Do we need quotas to ensure that women can continue to climb the corporate Ladder fairy as they

balance work and family?

“Personally, I don’t like quotas,” Reding said recently. “But i like what the quotas do.” Quotas

get action: they “open the way to equality and they break through the glass ceiling,” according to Reding,

a result seen in France and other countries with legally binding provisions on placing women in top

business positions.

I understand Reding’s reluctance-and her frustration. I don’t like quotas either; they run counter to

my belief in meritocracy, government by the capable. Bur, when one considers the obstacles to achieving

the meritocratic ideal, it does look as if a fairer world must be temporarily ordered.

After all, four decades of evidence has now shown that corporations in Europe as the US are evading

the meritocratic hiring and promotion of women to top position— no matter how much “soft pressure ”

37

2010——2019年考研英语二真题及答案

is put upon them. When women do break through to the summit of corporate power--as, for example,

Sheryl Sandberg recently did at Facebook—they attract massive attention precisely because they remain the

exception to the rule.

If appropriate pubic policies were in place to help all women---whether CEOs or their children’s

caregivers--and all families, Sandberg would be no more newsworthy than any other highly capable person

living in a more just society.

36. In the European corporate workplace, generally_____.

[A] women take the lead

[B] men have the final say

[C] corporate governance is overwhelmed

[D] senior management is family-friendly

37. The European Union’s intended legislation is ________.

[A] a reflection of gender balance

[B] a reluctant choice

[C] a response to Reding’s call

[D] a voluntary action

38. According ti Reding, quotas may help women ______.

[A] get top business positions

[B] see through the glass ceiling

[C] balance work and family

[D] anticipate legal results

39. The author’s attitude toward Reding’s appeal is one of _________.

[A] skepticism

[B] objectiveness

[C] indifference

[D] approval

40. Women entering top management become headlines due to the lack of ______.

[A] more social justice

[B] massive media attention

[C] suitable public policies

[D] greater “soft pressure”

Part B

Directions:

You are going to read a list of headings and a text. Choose the most suitable heading from the list A-F

for each numbered paragraph (41-45).Mark your answers on ANSWER SHEET1. (10 points)

[A] Live like a peasant

[B] Balance your diet

[C] Shopkeepers are your friends

38

2010——2019年考研英语二真题及答案

[D] Remember to treat yourself

[E] Stick to what you need

[F] Planning is evervthing

[G] Waste not, want not

The hugely popular blog the Skint Foodie chronicles how Tony balances his love of good food with

living on benefits. After bills, Tony has ?60 a week to spend, ?40 of which goes on food, but 10 years ago

he was earning ?130,000 a I year working in corporate communications and eating at London's betft

restaurants'" at least twice a week. Then his marriage failed, his career burned out and his drinking became

serious. "The community mental health team saved my life. And I felt like that again, to a certain degree,

when people responded to the blog so well. It gave me the validation and confidence that I'd lost. But it's

still a day-by-day thing." Now he's living in a council flat and fielding offers from literary agents. He's

feeling positive, but he'll carry on blogging - not about eating as cheaply as you can - "there are so many

people in a much worse state, with barely any money to spend on food" - but eating well on a budget.

Here's his advice for economical foodies.

41._____________________

Impulsive spending isn't an option, so plan your week's menu in advance, making shopping lists for

your ingredients in their exact quantities. I have an Excel template for a week of breakfast, lunch and dinner.

Stop laughing: it's not just cost effective but helps you balance your diet. It's also a good idea to shop daily

instead of weekly, because, being-human, you'll sometimes change your mind about what you fancy.

42____________________________________________________________

This is where supermarkets and thci; anonymity come in handy. With them,

there's not the same embarrassment as when buying one carrot in a little

greengrocer. And if you plan properly, you'll know that you only need, say, 350g

of shin of beef and six rashers of bacon, not whatever weight is pre-packed in the

supermarket chiller.

43_________

You may proudly claim to only have frozen peas in the freezer - that's not

good enough. Mine is filled with leftovers, bread, stock, meat and fish. Planning

ahead should eliminate wastage, but if you have surplus vegetables you'll do a

vegetable soup, and all fruits threatening to "go off' will be cooked or juiced.

44___________________________________

Everyone says this, but it really is a top tip for frugal eaters. Shop at butchers,

delis and fish-sellers regularly, even for small things, and be super friendly. Soon

you'll feel comfortable asking if they've any knuckles of ham for soups and stews,

or beef bones, chicken carcasses and fish heads for stock which, more often than

not, Theyil let you have for free.

45__________________

You won't be eating out a lot, but save your pennies and once every few

39

2010——2019年考研英语二真题及答案

months treat yourself to a set lunch at a good restaurant - ?1.75 a week for three

months gives you ?21 - more than" enough for a three-course lunch at

Michelin-starred Arbutus. It's ?16.95 there - or ?12.99 for a large pizza from

Domino's: I know which I'd rather eat.

Section III Translation

Directions:

Translate the following text from English into Chinese. Write your translation on ANSWER SHEET 2.

(15 points)

I can pick a date from the past 53 years and know instantly where I was , what happened in the news

and even the day of the week. I’ve been able to do this since I was four.

I never feel overwhelmed with the amount of information my brain absorbs my mind seems to be able

to cope and the information is stored away reatly. When I think of a sad memory, I do what everyone does-

try to put it to one side. I don’t think it’s harder for me just because my memory is clearer. Powerful

memory doesn’t make my emotions any more acture or vivid. I can recall the day my grandfather died and

the sadness I felt when we went to the hosptibal the day before. I also remember that the musical paly

Hamopened on the Broadway on the same day- they both just pop into my mind in the same way.

Section IV Writing

47. Suppose your class is to hold a charity sale foe kids in need of help. Write your classmates an

email to

1) inform them about the details and encourage them to participate .

2) Don’t use your own name, use “Li Ming” instead. Don’t write your address.(10 points)

48 write an essay based on the following chart. In your writing, you should

1)interpret the chart and

2)give your comments

You should write about 150 words on the ANSWER SHEET .

试题解析

Section I Use of English

1. (However)

【解析】空前作者讲到“鉴于电子货币的优势,你也许会认为,我们将快速步入非现金社会,实现完全电子支付。”而空后说“真正的无现金社会很可能不会马上到来”这两句话语义是转折的,因此答案A。 B. moreover表递进 C. therefore 表结果 D. Otherwise表对比

2. (around)

【解析】由空格所在句的“but” 得知,句子前后是转折关系。事实上,这样的预测已经 二十年了,但迄今还没有实现。A. off 停止 B. back 返回 C. over 结束,与后文均不构成转折,故答案选D.

around出现。

3. (concept)

【解析】空格所在的句子意思为例如, 1975年《商业周刊》预测电子支付手段不久将“彻底改变货币本身的____”将四个选项带入,能够彻底改变的对象只能是金钱的概念(定义),而A“力量”,C40

2010——2019年考研英语二真题及答案

“历史”,D“角”,语义都不恰当,并且如果选择 role的话,应该是复数roles, 因为是金钱的作用不止一个,故答案选B。

4. (reverse)

【解析】空格填入的动词跟前面的动词 revolutionize (变革)意思上应该是同义替换的,要选择含有变革,彻底改变意思的词汇,四个选项中A. reward 奖励 B. 抵抗 C. resume 重新开始,继续,都不合适,只有D选项reverse“颠覆”最为贴切,本句译为“电子支付方式不久将改变货币的定义,并将在数年后颠覆货币本身。”

5. (slow)

【解析】根据前面的句意得知,早在1975年就预测了无现金社会将到来,而实际上作者讲到“真正的无现金社会很可能不会马上到来”,因此也得出这种变革是一个缓慢的过程,故答案选择C。 A.

silent沉寂的,B. sudden突然的,D. steady稳定不变的。

6. (against)

【解析】上一段末句提出本段的论点,即人们进入无现金时代的速度缓慢的原因。因此本段应围绕纸币系统不会消失来阐述。而且由句首的Although得知,空格所在句与前一句是转折关系。尽管电子支付手段可能比纸币支付方式更加高效,然而以下几个方面解释了纸币系统“不会”消失的原因,故答案选B,work against妨碍,对…产生消极影响。A. work for 为…而工作 C. work with 与…共事,对…起作用 D. work on 从事…工作,对…起作用,都不合适。

7. (expensive)

【解析】本句陈述的原因都是关于上句提到的传统支付方式的优点,即推广电子支付方式不利之处。所以根据这个基调,得出选项productive不对,最后根据空后的内容推理出消极意思的选项expensive,其他选项意思放到空格处不合理,imaginative,意思是“虚构的、富于想象力的”;sensitive,意思是“敏感的、容易受伤的”。故本题正确答案为B。

8. (dominant)

【解析】空格所在句译为...使得电子货币成为____支付方式,将四个选项带入,C, D是比较恰当的,再结合本文章的主旨,应该选择“占主导地位的,支配地位”这层意思的D选项。A. similar

相似的B. original原始的,独创的,都不合适。

9. (provide)

【解析】 纸质支票支付能够____收据,这是和电子支付相比的一大优势,A. collect 收集收据,C. copy 复印收据,D. print打印收据都和实际生活不符合。应该是B. provide提供收据。

10. (give up)

【解析】该动词短语的宾语是前文的something, 指代上文的advantage,纸质支票支付能够提供收据这一优势,肯定是消费者不愿放弃的。和优势相搭配的动词短语不能是B. take over接管,也不能是C. bring back拿回来,D. pass down传递、遗传也不符合。A. give up放弃一种优势,符合语境,为正确答案。

11. (before)

【解析】这里考查的是时间连词的应用。句子意思是“在支票兑换成现金之前要花上好几天”,符合句意的只有before,其它三项都不符合。

12. (withdrawn)

41

2010——2019年考研英语二真题及答案

【解析】这里考查动词辨义。原文句子意思是“资金是从发卡机构的账户里提取的”,withdraw有“提款、取款”的意思,这里是指纸币从银行账户中“被取出”故为正确答案。

13. (Because)

【解析】这里考查的是连词的应用。从原文可以看出空后的两个句子在意思上存在着因果关系,“因为电子支付是即付的,所以消除了客户的付款”。四个选项中只有C because可以表因果,其他三项均不能表因果。故答案为C。

14. ( raise)

【解析】 这里考查的是动词辨析以及上下文语义衔接。[A] hide “隐藏,隐瞒”,[B] express “表达,表示”,[C] raise “举起,提高,引发”,[D] ease “减轻,缓和”,四个选项中能和concerns 构成搭配的只有raise,故正确答案为[C]。

15. .(stored)

【解析】这句讲了an unauthorized hacker has been able to access a computer database and to alter

information__________ there. “一些黑客入侵电脑数据库并且更改_____信息”根据空前信息可知是入侵电脑数据库,所以information 是被储存在电脑数据库中的信息。

16. .(uncommon)

【解析】此题考查一致性。空格所在句“The fact that this is not an__16_occurrence means that…”中this指代上文中that从句的内容,即黑客能够获取电脑数据库和更改储存的信息。因此not

an_16_occurrence应该能体现这一行为的特征,而上文提到“We often hear media reports that…”,其中的often正是对这一行为的特征解释,即not an__occurrence等于often的含义,对比选项,只有C选项uncommon符合,带入后意为“经常发生的事情”。

17. (steal)

【解析】本题缺少谓语动词,通过语法结构可以看出,主语是dishonest persons,并通过后面的其他人的帐户,可以推定为答案是负向的,只有A steal符合题意,语义上也说得通,故为正确答案。

18.【答案】B.(prevention)

【解析】文章最后一段首句谈论电子付费方式的又一个缺陷:会引起安全和隐私问题。接下来就开始解释这个现象。空格所在句提到“对这种欺诈的_18__绝非易事,而且一个新的电脑科学领域正在形成来_19__安全问题。”因此,本句在谈论对问题的解决应对。18空格与19空格所填内容语意上应该是一致的。浏览选项,18空只能选prevention,即防止这种欺诈行为发生并非易事,而C选项manipulation是“操纵”的意思,D选项justification意为“解释,证明……合理”,均不合理。

19. .(cope with)

【解析】此空格解释同18空格,应选有“处理,解决”意思的选项,只有A选项cope with合适。B选项fight against意为“对抗,抵制”,而宾语是security issues,因此不符合。

20. .(trail)

【解析】此空所在句提出了使用电子付费方式的又一个担心,即会留下__20_,空格后的定语从句解释了空格内容,即它包含大量个人数据。浏览选项,只有trail符合,意为“痕迹”。B碎片从语义上均说不通,C路径有一定的干扰性,但相比较D而言,痕迹更为合适,故为正确答案

Section II Reading Comprehension

Part A

42

2010——2019年考研英语二真题及答案

Text 1

21.

【解析】第一段第二行指出笑话是关于纺织厂自动化程度的,后一句具体说明了笑话的内容:工厂平均每天只有两个人,一人一狗。人的工作是喂狗,狗的工作是看机器,暗示了工厂所有的生产工作都是由机器自动完成的。因此,这个笑话是用来说明技术进步的影响,故选A。

22.

【解析】 事实细节题,通过题干“根据第3段,要想成为一个成功的雇员,一个人得……”, 我们首先可以定位到文章第三段,由第三段的最后一句话“Therefore, everyone needs to find their

extra-their unique value contribution that makes them stand out in whatever is their field of employment.”意思是“因此,人人都需要有另外的价值,异于常人的独特价值能够让他们在各自的雇佣市场上脱颖而出。”,我们可以得出,题干中“to be a successful employee”与第三段的最后一句话中的“that makes

them stand out in whatever is their field of employment”是同义替换,“everyone needs to find their

extra-their unique value contribution”与D选项中的“contribute something unique”是同义替换,所以D选项正确。

A、B两个选项与第三段的倒数第二句话意思不符,是干扰选项。C选项与第三段的第一、二句话意思相反,重点关注的是“But ,today ,average is officially over.”,意思是“现如今,拥有一般水平不行了。”

23. 【答案】B

【解析】根据题干定位到第四段,第一句technology has been eating jobs(技术使工作机会减少)也反映了该段的主旨。而根据题干quotation一词,我们读到引号里有“shed workers (解雇工人)”、“roughly one out of every three manufacturing jobs-about 6 million in total -disappeared (大约1/3的手工生产工作机会消失,总计6百万)”,可推断出B选项。

24. 【答案】B

【解析】细节题。根据题干reduce unemployment减少失业,可以定位到文章中最后一段,这段出现了与之类似的表达“support employment” 促进就业,而题干表述“the most important”与文章“nothing would be more important than”相对应,指出促进就业最重要的是颁布类似于“”的法案来保障人们接受高等教育的权利,选项C与之吻合,故正确。A项加速信息技术产业变革,C项促进经济全球化,均未提及,故排除。D项是干扰项,虽提及要颁布更多法案,但颁布法案的目的实际是为了保障教育,故也排除。

25.

【解析】该题是主旨大意题,主要考查考生根据文章内容凝练主旨大意的能力。从整个文章的脉络来看,第一段以亚当?大卫森一篇论文中关于现代工厂自动化与仅需要一人一狗两个员工的一则笑话,揭示了科技进步给人们带来的影响。第二三段是科技的进步引起工厂自动化水平提高,普通员工如果没有竞争力和突出优势,就很容易失去工作,因此也对员工提出了更高的要求(extra-unique

value contribution)。第四段就是员工只有不断地提高自己的教育水平,才能让自己脱颖而出(to have

more and better education to make themselves above average)。最后一段点明主题,average is officially

over。由此可见,全文一直在围绕这一宏观主线展开,这一主线也统领全文,所以正确答案为C。

Text 2

43

2010——2019年考研英语二真题及答案

26. (stay in a foreign temporarily)

【解析】词义句意题。根据题干,首先定位到首段末句。在这句中,birds of passage是前面一句中的1/4的意大利移民的昵称,他们只在美国居住了一段时间,但最终还是返回意大利。A项的内容在首段首句有提及,但是它突出强调的是横跨大西洋的移民,这也是一世纪前的情况,而如今birds of passage可能来自世界各个角落,并非局限于大西洋两岸。B项与段意不符,D项文中未提及。

27. (should be adopted to meet challenges)

【解析】推理判断题。根据题干,直接定位到第二段。解题关键可定位到“…, but we need to change

the way we think about categories. We need to look beyond stick definitions of legal and illegal. To start,

we can recognize the new birds of passage, …We might then begin to solve our immigration challenges.”大意是“我们需要改变的是关于分类的思考方式,突破合法和非法的严格限制。首先承认短暂移民者的存在,然后解决移民问题面临的挑战。”C项高度总结了以上几点。A项与原文意思不符。B、D项在文中未提及。

28. (the freedom to stay and leave)

【解析】事实细节题。根据题干,可定位到第三段。解题关键在于对“They prefer to come and go

as opportunity calls them .They can manage to have a job in one place and a family in another.”大意是“他们跟着机会走,来去自如。他们可以在一个地方立业,在另一个地方成家”。强调的是工作机会,而不在乎工作地点。D项是这句意思的高度概括。A项是对原文的片面理解,吸引短暂移民者的不仅仅是来自金钱的激励(financial incentives),还有工作机会和工作理念。B项在文中未提及。C项中的regular jobs(一般工作)在文中未提及,也是对文意的曲解。

29. (with legal tolerance)

【解析】推理判断题。根据题干,可定位到第五段。题干问到“根据作者的意思,我们(美国)应该怎样对待这些短暂移民者?”在本段中,作者写道我们应该“Looking beyond the culture war logic

of right or wrong means opening up the middle ground and understanding that managing immigration today

requires multiple paths and multiple outcomes. Including some that are not easy to accomplish legally in

the existing system”,大意是“我们应该超越移民合法性方面的文化之争,重现看待中间地段,充分意识到当今的移民管理体系需要各种途径,从而取得多样化的结果,来解决现今移民体系下用法律手段很难解决的问题”,暗含了C项中的tolerance也就是对multiple paths and multiple outcomes的改写。A、B和D项在文中未提及。

30. (legal or illegal: big mistake)

【解析】主旨大意题。文章第二段第二句中提到“我们把新移民分成两类:合法移民和不合法移民”,同时作者认为“我们不需要局限于合法与不合法这样严格的定义”,这说明了从合法和不合法角度对于移民的分类是错误的。另外,文章最后一段最后一句“包括在现行的移民体系中不容易合法的实现的一些事情”也反映了文章的中心。即,从合法和不合法角度对于移民的分类是错误的。故选D(合法或非法:大错误)。 A项谈到短暂移民者的移动是个错误,偏离了文章主旨。B和C项说的是risk(风险),文中并未提及,偏离文章主旨。

Text 3

31.【答案】 [D] predetermine the accuracy of our judgment

【解析】细节题。题干问的是“作决定过程中所需的时间可以_____”。文章第一段提到“如果44

2010——2019年考研英语二真题及答案

我们在做出反应之前花点儿时间来思考,那么将会减少甚至消除我们快速反应所带来的负面影响”,也就是说我们做决定所花的时间决定了我们判断的准确性。文章第二段第二句话也隐含本题正确答案线索。第二句以But这一转折连词引导,应该重点关注其后表达的信息,“但是,我们需要更多的时间来评估其他要素。”而本段的第三、四句子,很明显地揭示出本题正确答案,尤其是第三个句子中的“accurately” 一词。选项D中的表达“可预先决定判断的准确性”,此外,此选项中的“accuracy”

为“accurately ”的同词异形,故此项为正确答案。选项A表达“依形势紧急性而定”错在无中生有,本文并没有出现类似信息;选项B“证明大脑反映的复杂性”,也是无中生有;选项C “取决于评估的重要性”,也与原文不符合,故排除。

32.【答案】[A] can be associative

【解析】细节题。题干问的是“我们对于快餐商标的反应速度表明决定是_____样的”,由题干的“fast-food logo”我们可以定位到第三段。第二段说处理人际关系问题时人们会仓促决定,第三段开头就说了,让人做出仓促决定的刺激因素不仅限于人际关系范围内。紧接着一句说人们对快餐商标的反应速度比一般阅读速度快。下一句阐述了原因:因为人们无意识地(unconsciously)将“快餐”与“速度”和“心急”联系在一起,并将这些冲动付诸行动。A说决定是有联系性的,正确,因为人们将“快餐”与“速度”,“心急”联系在了一起。B说决定是无意识的,与原文意思相反,错。C说决定是危险的,原文未提及,排除。D说决定是不冲动的,与原文意思相悖,故排除。

33.【答案】[C] think before we act

【解析】细节题。先看题干说“为了逆转仓促决定所带来的影响,我们可以做_____”, 根据题干定位到第四段。第四段通过两个例子说明我们应该怎样克服负面影响,第一个例子表示“如果我们会对消费产品或者房产选择做出“过度反应”,我们可以在购买之前先思考一会儿”,由此可说明我们应该在行动之前先思考来消除负面影响,因此选择答案C。其他选项:A项“相信我们的第一印象”;B项“按照人们通常所做的去做”;D项“征求专家意见”均不符合题意。

34.【答案】[D] adequate information

【解析】细节题。题干问的是“John Gottman认为可靠的快速反映是基于_____的。”由题干John

Gottman 定位到全文倒数第二段。其中第一句:John Gottman, the marriage expert,the marriage

expert,explains that we quickly”thin slice”information reliably only after we gound such snap reactions

in ”thick sliced”long-term study.婚姻专家约翰.古德曼解释说,我们快速反应的信息的可靠性是建立在这样的快速反应的行为是以长期的研究为基础而做出的快速反应行为。其中gound是题干中base

on 的同意置换,long-term study长期的研究与D选项adequate information相互呼应。由此可判断出[D] adequate information(足够的信息)是本题正解。该段第二句话是When Dr,Gottman really wants to

assess whether a couple will stay together, he invites them to his island retreat for a much longer

eveluation,two days ,not two seconds.当古德曼博士想去评估一对夫妻是否应该继续在一起时,他会邀请他们到他的岛上进行一个更为长期的调查,是两天而不是两秒。第二句是对第一句的举例说明,更加验证此选项。

35.【答案】[C] optimistic

【解析】态度题。根据35题题干reversing the high-speed trend是全文的最后一句,所以解此题可先定位到全文的最后一段。最后一段最后两句:Although technology might change the way we react,

it hasn’t changed our nature. We still have the imaginative capacity to rise above temptation and reverse

45

2010——2019年考研英语二真题及答案

the high-speed trend.译为:尽管技术可能改变我们反应的方式,但是它并没有改变我们的本性。我们仍然有能力去克服诱惑并扭转这种高速度的趋势。由此我们可以看出作者的态度是非常确定的,因此B选项uncertain(不确定)首先排除;We still have the imaginative capacity…表面作者对于我们的能力是有信心的。因此[C] optimistic(乐观的)是正解。A选项tolerant(容忍的) 、D选项doubtful(怀疑的)在原文中没有体现,属于无中生有的选项。

Text 4

36. men have the final say

【解析】根据题干corporate workplace定位到第一段。首句就说欧洲性别不平等,in particular进一步指出corporate workplace,说明工作中性别尤其不平等。紧接着说欧洲公司高管职位remain

overwhelmingly male,说明以男性为主导。Indeed进一步解释,指出女性在欧洲公司董事会只占有14%的席位。所以,B选项是基于首段信息给出的推论。A项women take the lead和D项senior

management is family-friendly都与原文相反;C选项是对文章overwhelm这个词出的干扰项。

37. a reflection of gender balance

【解析】根据题干,定位到第二段首句,intended legislation是对is now considering legislation的同义改写。该句意思为“欧洲国家现在考虑立法来迫使公司董事让妇女的比例达到60%”,因此立法是为了保持性别的平衡。B选项的reluctant是对第5段的Reding’s reluctance出的干扰项,并不是说European Union的立法。C选项a response to Reding’s call不正确,Reding号召的是voluntary

action, D也是干扰项,而真正的立法缘由是对gender balance的反思,所以A项正确,也是文章中心的反映。

38. get top business positions

【解析】定位至第4段,Reding说自己不喜欢quotas,后面出现了but,他真正的观点在but之后,他说他喜欢quotas所做的事情,即get action,后面的冒号是对get action的解释。核心的答案在a result seen in France and other coutries with legally binding provisions on placing women in top

business positions。a result是前面内容的同位语,进一步补充说明,所以选A。B项see through the glass

ceiling是对原文break through the glass ceiling的望文生义,属于肤浅选项,也和原文意思不符。C和D选项属于无中生有。

39. approval

【解析】本题问的是作者对Reding的呼吁的态度。Reding的appeal最早出现在第2段,即呼吁在董事会中有40%的女性,以实现性别均衡。而第四段再一次提到Reding 的观点即“他自己也不喜欢quotas,但是quotas本身确实起到了作用”;接着作者在第五段给出了自己的观点,先是说可以理解Reding,自己本身也不喜欢quotas,但是“既然现在meritocratic ideal(精英管理的理想)有障碍,确实需要一种强制的手段,即强制设定男女比例。”所以可以看出作者是持“赞成”的态度。

40. suitable public policies

【解析】题干中的women entering top management become headlines是对第6段第二句话when

women do break through to the summit of the corporate power的同义改写,become headlines是对后面for example所举的Sheryl Sandberg的事例的概括。答案出现在第7段开头。第7段是提出一种解决措施,“If appropriate pubic choices were in place to help all women, ...Sandberg would be no more

”,这个句子是if虚拟条件句,是对未来的一种美好展望,也是提出观点的一种方式,46

2010——2019年考研英语二真题及答案

意思是“如果有合理的公共政策来帮助所有的女性,Sandberg也就没有报道价值了”。所以正确答案是C,因为缺少“suitable public policies”。

Part B

41.【答案】F Planning is everything

【解析】段落首句谈到“Impulsive spending isn’t an option, so plan your work’s menu ”,其表达的含义是:冲动消费不是一个好的选择,所以提前计划你一周的菜单,为你所需材料的具体数量做一个购物清单。首句中出现了因果逻辑关联词so,而下文又没有出现明显转折,因此首句是本段的中心句。文章进而提及作者为此专门做一个Excel表格,并且认为这样做不仅花钱少并且有助于均衡饮食。显然文章的中心在于首句谈到的“plan”,而中心不是选项B表达的“balance your diet”,故答案为F。

42.【答案】E Stick to what you need

【解析】段落第二句话中的代词“them”指代第一句中的“supermarkets and their anonymity”。该句通过这一指代顺接第一句,表达了一个否定的含义,即你不需要在小贩那里感到尴尬。紧接着第三句用肯定的语气指出“if you plan properly, you’ll know that you 350g of shin of

”即如果你合理的规划,你就会清楚知道你想要什么,比如你只需要350克牛肉。作者通过语义上层层递进的方式指出了这一段的中心:你需要什么就买什么,要对你所需要购买的东西的分量要坚持。因此选项E为正确选项。

43.【答案】G Waste not, want not

【解析】本段内容首先谈到“你可能骄傲的说冰箱里只有冷冻的青豆,但是这还不够”。这句话中虽然没有明显的转折词,但在语义上属于隐性转折,因此段落的重点应该在后面。第二句提到“Mine

is ”,其中Mine等于my freezer,通过指代顺接上一句话。接着第三句前半句指出“提前做好计划可以避免浪费”,后半句具体陈述了怎样避免浪费。其中“eliminate wastage”与选项G中的“waste not”构成同义替换。虽然该句中出现了planning, 但是本段的主要内容是谈到对于剩余的食物要尽可能充分利用,从而避免浪费。所以选项G谈到“不浪费,不愁缺”为正确选项。

44. Shopkeepers are your friends

【解析】该段首句的句内出现转折,重点在转折之后。第一句but转折之后提到“it really is a top

tip”,即这真的是一个好的提议。那么首先要还原it所指代的内容。句前没有提供信息,句后第二句提到“shop at butchers,...regularly, ..and be super friendly”。其中“be friendly”通过词性转换和选项C中的“are your friends”够成同义替换。同时,根据就近指代原则,这也是it所指代的内容。最后一句通过具体的信息描述了购物时表现出友好的态度所带来的好处:they will let you have for free(通常他们都会免费给你),因此选项C为正确选项。

45. Remember to treat yourself

【解析】该段首句句内出现转折,重点在转折之后。第一句but后提到“save your pennis and once

every few months treat yourself to a set lunch”,其表达的含义是要节省钱,但可以每几个月款待自己一次。而该句也是本段落的中心句。段落余下的信息都是在用数据来解释这个道理。选项D中出现了“treat yourself”,属于原词复现。因此选项D为正确选项。

Section III Translation

46. 翻译

47

2010——2019年考研英语二真题及答案

参考译文

从过去的53年间任选一天,我能立刻回想起当时我身在何方,当天新闻中发生何事,甚至那天是周几。自从四岁,我就具备这种能力。

我从不会因大脑吸信息量过大而感到难以承受。我的大脑似乎可以处理它们,并将其有序地存储于脑中。每当忆及忧伤往事,和其他人一样,我会尽量将其搁置一旁。我不认为因为我的记忆更为清晰,自己就比其他人更难做到此事。好记性并没有让我的情感体验更鲜活生动。祖父去世那天的情景和之前那天我去医院看望他时的伤心欲绝都历历在目。我也还记得当天在音乐剧《毛发》百老汇开场演出。这两件事都以同样的方式跃入我的脑海。

【解析】

1. I can pick a date from the past 53 years and know instantly where I was, what happened in the news

and even the day of the week.

【词汇】instantly:立即、马上;

【分析】句子主干为I can pick a date…and know…;宾语部分为where…,what…and even…。句子中并列结构突出。

【译文】从过去的53年间任选一天,我能立刻回想起当时我身在何方,当天新闻中发生何事,甚至那天是周几。

2. I’ve been able to do this, since I was four.

【分析】句子主干为I’ve been able to do this, since 引导时间状语从句

【译文】自从四岁,我就具备这种能力。

3. I never feel overwhelmed with the amount of information my brain absorbs.

【词汇】overwhelmed:压垮,压倒,淹没; absorb:吸收

【分析】句子主干为I never feel overwhelmed with…,省略关系词的定语从句my brain absorbs作后置定语修饰information

【译文】我从不会因大脑吸信息量过大而感到难以承受。

3. My mind seems to be able to cope and the information is stored away neatly.

【词汇】cope:处理; store:存储; neatly:整齐的,整洁的

【分析】句子主干为My mind seems to be…and the information is…

【译文】我的大脑似乎可以处理它们,并将其有序地存储于脑中。

4. When I think of a sad memory, I do what everybody does---try to put it to one side.

【词汇】think of:考虑,想起;

【分析】句子主干为I do what…what引导宾语从句,破折号后try to put it to one side对其进行解释说明;when引导时间状语从句。

【译文】每当忆及忧伤往事,和其他人一样,我会尽量将其搁置一旁。

5. I don’t think it’s harder for me just because my memory is clearer.

【词汇】clear:清晰的

【分析】句子主干为I don’t think its harder for me, because引导原因状语从句。It指代上句话内容。

【译文】我不认为因为我的记忆更为清晰,自己就比其他人更难做到此事。

48

2010——2019年考研英语二真题及答案

6. Powerful memory doesn’t make my emotions any more acute or vivid.

【词汇】powerful:强大的,权力大的; emotion:情感,情绪; acute:敏锐的,敏感的; vivid:生动的,形象的

【分析】句子主干为Powerful memory doesn’t make…

【译文】好记性并没有让我的情感体验更鲜活生动

7. I can recall the day my grandfather died and the sadness I felt when we went to the hospital the day

before.

【词汇】recall:回忆起,回想起

【分析】句子主干为I can recall the day…and the sadness…省略引导词的定语从句my grandfather

died作后置定语修饰the day;同样省略引导词的定语从句I felt修饰sadness(定语从句关系词做宾语时可以省略);when引导时间状语从句对the sadness I felt进行修饰。

【译文】祖父去世那天的情景和之前那天我去医院看望他时的伤心欲绝都历历在目

8. I also remember that the musical play Hair opened on Broadway on the same day---they both just

pop into my mind in the same way.

【词汇】musical play:音乐剧; Hair:《毛发》(1968年上演,是对美国百老汇音乐剧的颠覆,获得托尼奖); Broadway:百老汇; pop:突然出现

【分析】句子主干I also remember that…,that 引导的宾语从句主干为the musical play Hair

opened…;破折号后they指代前两句话的内容,表示同一天发生的两件截然不同的事件,突出前面提到的好记性并未使我的情感体验更鲜活生动。

【译文】我也还记得当天在音乐剧《毛发》百老汇开场演出。这两件事都以同样的方式跃入我的脑海。

Section IV Writing

47. 应用文范文

Dear my beloved classmates,

On the evening of January 4th, 2013, we will hold a charity sale for children who need help at the

school auditorium.

The kids come from remote areas where they can not be educated properly. Our assistance may change

their destinies. Many pop stars, such as Jay and Jackie Chan, will attend the activity. The school master and

most of the teachers of our school will also join us.

I trust you will be disengaged and able to give the poor children a hand. Thank you very much for

your kindness.

Yours sincerely

Li Ming

48. 大作文写作

Emerging from the bar-chart above is a popular phenomenon that the proportion of the students having

par-time jobs has changed during the four years’ study. The proportion increases slightly from the first

year to the third year, however, the fourth year has witnessed a fast increase, surging to 88.24%.

As they are about to enter into the society, more people think that concurrent post after school benefits

49


本文发布于:2024-09-21 17:28:51,感谢您对本站的认可!

本文链接:https://www.17tex.com/fanyi/27151.html

版权声明:本站内容均来自互联网,仅供演示用,请勿用于商业和其他非法用途。如果侵犯了您的权益请与我们联系,我们将在24小时内删除。

标签:选项   意思   移民   工作   答案   真题   解析   飞机
留言与评论(共有 0 条评论)
   
验证码:
Copyright ©2019-2024 Comsenz Inc.Powered by © 易纺专利技术学习网 豫ICP备2022007602号 豫公网安备41160202000603 站长QQ:729038198 关于我们 投诉建议